Sunteți pe pagina 1din 184

* C£OM C£OM CEDM CEDM CEDM C£OMCEDM CEDM CEDM CEDM C£OM CE *

® ~

~ ~

~ Study Material forCompetitive Examinations

o ~

iu ~

; ~

§i ~

~ ~

.0 TEST OF REASONING ...

§ ~

§ ~

~ ~

u ~

~I ...

§ ~

~ ~

B ~

~ ~

u ~

§ ~

U~ :

CENTRE FOR EDUCATIONAL DEVELOPMENT OF MINORITIES ~

~ Osmania University

Ufil (Sponsored by the Minorities Welfare Department, Govt. of Andhra Pradesh) ~

Nizam College Campus, Basheerbagh, Hyderabad - 500 001 ...

@ ~

§ ~

* CEDM CEDM CEDM CEDM C£OM CEDM CEDM C£OMC£OM C£oM CEDM CE *

Study Material for Competitive Examinations

TEST OF REASONING

By

D. Subramanyam

M_ COlli Resource Person for Civil Scrviccc and other COIllPCl1tJ"C Ex.unuunions

CENTRE FOR EDlICATIONAL DEVELOPMENT OF MINORITIES Osmania University

(Sponsored by Minorities Welfare Department. Gov! of A. P.) Nizam College Car-pus. Basheerbagh. Hyderabad - 500 001.

Phone No: 23210316

CONTENTS

2. 3 4 5. 6. 7. 8. 9 10. II. 12 13 14. 15. 16. 17. 18. 19. 20. 21.

Numbers Series Missing Numbers Number Analogy .. Odd Numer ... 1 Letter Series. Alphabet Series .. Alphabet Analogy .. Coding, Decoding ...

Blood Relations Directions Analytical ..

Quantitative Syllogism.

Strong and Weak Arguments .. Implicit Assumptions .... Courses of Action." Statements and Conclusions .. Common Sense ....

Degree of Truth """""""''''',,.,,'''''''',,.

Data Sufficiency.i.v,.. .. "." .. ",,'"

Answers.i..... """"'"''''''''''''''''''''''''''

" ".",11 to 15 ".16 to 17 ."."",18 to 20 .. 21 to 22 ".""."".23 to 25 . ... , .. ,,26 to 28 ,,29 to 30 . .... " .... ,31 to 48 ............ .49 to 54 , ..... , .... 55 to 61 .... ,.,.,62 to 105 .... ",.106to 119 .. 120 to 126 . ... 127 to 129 ,,130 to 141 ,,, ..... 142 to 150 ".151 to 154 ..155 to 160 """""""'" " ... " .. 161 to 163

..." .... "" .. 164 to 168 ",,,.,,,,,,,,, 169

PREFACE

Taking competitive examination has become the order of the day for any educated young man who aspires to capture any coveted position in civil services. banking services, market sector or is desirous of seeking a seat in any prestigious college. The approach required for any job seeking competitive examination is different from that of taking an academic examination for securing a degree in any course. A job aspirant should possess three qualities - General ability, Intellectual ability and Personal quality or individual outlook One has to acquire the knowledge of hi. surroundings, physical as well as social with a inquisitive mind, to know at least something about everything i.e. History, Polity, Economy, Geography, Science and Current Affairs. Ability to analyse an issue or a problem with clarity in thinking and writing, with logical expression and also to offer a feasible solution to the problem. Good personal qualities to lead and to influence others actions are very essential for such competitive examinations.

The above qualities are not innate or inborn. they can be acquired, and cultivated through intensive coaching. Coaching develops knowledge. skills and qualities Thus, coaching is nurturing personality. There are two ways of offering coaching - Personal and Postal

In order to improve the participation and performance of the candidates belonging to minorities in such competitive examinations, Minorities Welfare Deparment, Government of Andhra Pradesh sponsored a project to Osmania University The University in turn established the Centre for Educational Development of Minorities' at Nizam College, and two Regional Centres- one at Osrnania College, Kurnool and the other at Andhra Muslim College, Guntur to provide personal and Postal coaching programmes and to offer Career Guidance to minority candidates in Andhra Pradesh.

The Centre for Educational Development of Minorities, Osrnania University, Hyderabad has been offering coaching programme for the benefit of the candidates belonging to Minority Communities appearing for various competitive examinations for the past Ten years at Nizam College and other regional Centres.

As a result of the feedback received from the candidates it was felt that most of them do not fare well at these competitive examinations not because they lack in talents but because they can neither afford to join the private coaching classes nor could purchase the study materials required for such courses.

In order to bridge this gap, the Centre for Educational Development of Minorities, decided to compile essential study material to help such of the poor candidates. While preparing these materials, care has been taken to cover important topics i of the core subjects. In most cases, brevity has been observed to present the materials in nutshell in order to save the time of the candidates. These study materials are not ends in themselves but they are the means to and end. They show the direction in which the candidates should channelise their study programmes

It is needless to express that no originality could be claimed for the preparation and presentation of these study materials. Extensive use of the available source books has been made to benefit the candidates to a larger extent.

We wish to place on the record the pains the compiler has taken to summarise and arrange the important materials for students. The Centre gratefully acknowledges services

Any concrete suggestions for the improvement of these materials will receive our highest appreciation.

[f these study materials arc of any help to the candidates. we feel immensely rewarded for the humble efforts we have put in.

We earnestly seek your cooperation in fulfilling the objectives of improving the participation and performance of minorities in various competitive examinations for which the government has placed funds at our disposal

Hvderabad

Dated: 13th June 2005

Prof M. MAQBOOL DIRECTOR C.E.D.M.

TEST OF REASONING

Mental Ability

Mental ability is a complex concept which includes the ability to assimilate facts, reason logically and quickly, and get to the root of a problem without being diverted by secondary issues. Mental a:;;,ty tests are de~igned and applied to test one's ability in applying his or her intellect in various areas -like comprehension. memory. reasoning, analysis, evaluation and judgement within these broad parameters one may have a sharper aptitude for one field than for another, 'such as dealing with figural relations or with numbers and words as symbols (Verbal and non-verbal ability in other words). However a general minimum aptitude is expected from everyone in all these fields.

Under Mental Ability we wil be discussing the major areas of mental ability like logical reasoning, numerical ability, verbal ability, non-verbal (figural) ability, quantitative aptitude data interpretation, etc .. which form the basis of general mental ability tests in any examination.

Though there is no definite rule in tackling these tests, we discuss in this chapter some broad principles, which if kept in mind, will nelp any body to go about them in the most efficient way and arrive at answers quickly and accurately.

VERBAL ABILITY: It is the ability to deal with letters, group of letters. words and group of words, etc., The questions to test verbal ability commonly consist of the following types.

a) letter series b) letter analogy c) letter classification

d) letter sequences e) word analogy f) word classification g) coding and decoding

Letter Series: These type of questions contain only letters of the al-

phabet placed in a specific pattern. It may initially appear rather difficult to understand the pattern followed, however, with some practice, we can find the pattern quite quickly.

When you come across a group of questions of the letter series type. quickly write down the alphabet and number the letters as below. Then give the numbers to the letters in the questlns and observe the pattern of the numbers. Then you can easily trace the missing letter.

1 2 A B

3 C

4 o

5 E

6 7 F G

8 9 10 11 H I J K

12 13 14 L M N

15 16 17 18 19 20 21 22 23 24 25 26 OPQRSTUVWXY Z

1

And if in any series lefters go beyond Z, then A comes again as 27th letter and B comes as 28th lefter and so on and so forth

Example

DFHJLNP?

For this given letter series you put numbers over each letter as shown below

468 D F H

10 12 14 16

J L N P ?

If you observe the numbers they are 4, 6, 8, 10, 12, 14, 16 which are changig in a regular manner I.e. each number comes by adding 2 to it preceeding number. So the missing letter's number should be 18 (i.e 16 + 2) and the letter Is 'R'.

Example: A 0 H M S ?

3 4 5 6 9

/\/\/\/\/\

1 4 8 13 19 26

By giving numbers to the letters it can be observed that the difference between successive numbers increases by 1 as we move to the right and hence the number of the missing letter should be 26 and the letter shall be 'Z'

Note: After 'Z' will come 'A' in the sequence because the counting is circular.

Letter analogies: In this type of questions. the question poi' and answer pair consist of letters. You have to examine the question pair and find the relationship between them and choose the answer pair that contains the same relationship as in the question pair. Some of the most common analogical relationships between letter pairs are

a) Direct alphabetic squential relationship

Ex : EF : IJ: :: NO" ?

o] SR b) ST c) GI d) XZ

Answer: (b) - Examine the question pair 'EF: IJ' They are In natural alphabetic order and so also 'NO'. There fore, the choice in Which natural alphabetic sequence Is present is 'Sf'.

b) Opposite alphabetiC relationship

Ex : FE : LK :: QP : ?

a) ST

bJUV

c)YX

d)VW

Answer (c) : It is clear that the letters in each term are in the reverse of alphabetic sequence hence look for on answer where letters are in the reverse alphabetic order.

c) Vowel/Consonant relationship:

Ex : AZY : EWV ::

a) GTS : KNM b) IUT : OBR c) ITU : OST d) IUT : URS

Answer :(b): Examine the question pair' AYZ : EWV; the first letters of each term are 'A' and 'E'. We will be on the right track if the first letter of the answer pair begins with the vowell and the second begins with the vowel O. This leaves us with the answer 0: choices 'b' and 'c'. Go bock to the question pair and look at the next two letters of each term.

Now examine the pattern in the second and third letters are arranged in the first and second terms. 'ZY' and WV' are in reverse order. This pattern cleariy eliminates choice 'c' leaving 'b' as the correct answer.

d) Skip letter relationship:

Ex AC: SD :: _ __

oj EG : FH b) EF: GH c) GH : KL d) HI : RS

Answer: (oj Examine the question pair AC : BD; The relationship is that the letters are in alphabetic order, with a letter skipped between terms. Therefore the answer choice is 'a' where o-Ietter is skipped in each term.

e) Letter form relationship :

This type of analogy depends on the form of the letter. Look for straight lines closed and open ends, circles and toils, etc.,

Example: I:T :: __ : __

a)X:T b)M:V c)L:N d)L:X

Answer: (c) Examine the question poi 'I: 1'. The frst term 'I' is composed of one straight stroke. The second term 'T' is composed of two straight strokes, The pattern is 1 :2.

Examine answer option 0) X: T : 2 strokes: 2 strokes - no pattern

Examine answer option b) M: V : 4 strokes: 2 strokes - no pattern

3

Examine answer option c) l: N : 2 strokes: 3 strokes - the pattern

is increaslg humber of strokes; 2: 3

Examine answer option (d) L : X 2 strokes: 2 strokes; no pattern

Hence (c) Is the answer as 'I' corresponds to the pattern of increasing number strokes of the question pars.

example:

DNIK : KIND .. __ _ __

a) STUDENT: TNEDUTS c) KNllB : BLINK

b) SHINK : KNIHS d) READ : DEAR

Answer (c) : Examine the GlUestlon pair 'BNIK : KIND'. The second term (KIND) of the question pair is made by reversing the order of the letters of the first term (DNIK). SimHarly 'KNILB' is reverse of 'BLINK'.

LETTER CLASSFICATION (FINDING THE ODD LETTER OR LETTER GROUP)

Ex: 1. A E IOU Y in this the odd letter is 'y' because it is not a vowel.

Ex: 2. ACE G J K M in this ietter series the odd one is J because after A are letter (B) is skipped and C has come next '0' is skipped E has come. F is skipped and E has come. Next if we skip' H' I shall come. Hence' J' is the odd letter.

Similarly there are various relationships which can exist between the items. Several types of relationships have been identified and covered below. We need not remember the names of these types. But we must understand the relationship and be able to solve the few exercises given with each type.

LETTER SEQUENCES: In this letters WIll be written in a sequential pattern and some of the letters will be taken out and those missing letter wil be given in one of the choices and we have to select the right choice which when placed fulfills the letter sequences pattern.

Examples 1) abc aabcaabbc aabbcc

2) abc bee cab abc bea cab abc

3) abe aobbcc ooobbbcec oaoobbbbcccc

4) abc eba abc ebo abc ebo abe

5) aeb cab acb cab oeb cob oeb cab

6) CIa bb ccoobbecaobbcc oobbccaabbcc

7) maa nbb mao nbb mao nbb mao nbb

8) oobb oobb cobb oobb oobb oobb,

*'

Word Analogy: There are several types of verbal reasoning tests which are usually asked in the competitive examinations. These tests require logical thinking. common sense reasoning and a reasonable knowledge of english language.

In this type of questions we are be given a .pair of words in capital letters followed by 5 choice pairs. We have to choose the choice that best expresses the relationship seen in the original pair.

a) Relationship based on position of letters.

Ex. a) AD

b)BE

c)CF d)DG

e)ER

Answer: (e) In each pair, two immediately following letters have been skipped. i.e. A(BClD, BICD)E ,ClDE)F, D(EF)G and it is not present in choice Ie) which is the odd pair.

b) Small and Capital letter relationship

Example a) AbC b) BcD c) CdE d) DeF e) Efh

Answer (e) : Each pair contains two capital letters and one small letter and choice (e) contains one capital letter and two small letters which is the odd one in the given problem.

c) Vowel - Consonant relationship

Ex : a) AbcdE b) Ifgho c) ApqrL d) UlmnE e) EwxyO

Answer (c) : Each group of letter consistss of two vowels one each in the beginning and the end. In between that two vowels, three letters in alphabetic sequence are given. But 1his is not the case with (c) Where the last letter is a consonant and hence [c] is the odd pair.

d) Leiter form relationship

Example: a) T

b)H

c) K

d) E

e) S

Answer: (e) All letters are formed by straight lines and choice e) has curved lines and hence odd one in the set

Verbal Classfication (odd one Out)

These question test your ability to observe differences and similarities among various objects. Verbal ClassflCation tests also known as 'Odd one Out' tests, consist of a list of 5-6 Items. These Items could be either words, group of letters, or flQures (digits), Out of the given 5-6 items, one is different from the others. In

6

other words. except one. all of them have some sort of similarity. You are required to study the given list of Items. determine the similarly between them and pick out the one which does not bear the some characterIStics as the others in the list. This different. unassoclated or odd item has to be picked out from the list of items given in the question.

Example: In the following five words. one Is different from the others.

0) Rode d) Walked

b) Flew c) Sailed

e) Travelled

Answer (e). If we study these five words we find that all of them are associated with 'movement' or 'travelling of a person. They all seem to be Interrelated. However. a careful analysis will reveal that the word (e) Travelled is different from all the others. The words (a) to (d) are specific ways of travelling ond as such the wod (e) TRAVELLED does not fit In the list. It is the odd word in this list of 5 words. 'This type of word or 'items' analysis is called 'Classification' or finding the 'odd-one out'.

fiNDING THE ODD WORD

These questions are like the above example. There are several basic relationships that could exist between words. As in the case of word analogy (section 2.1.2) questions the ftst step is to identity the relationship. It would be useful if you quickly reVTew section 2.1.2 before going ahead. Several types of relationships have been identified and covered below. You do not need to remember the names of these types. You must understand the relationship and be able to solve the few exercises given with each type.

Relationship Based on Meaning

Example a) Different b} Separate

d) Similar e) Distinguishable

c} Distinct

Answer: (d). All the other words are synonyms.

Inter-relatlonshlp of Words

Example

0) Mars b) Mercury c) Jupiter d) Challenger

e) Neptune

Answer: (d) All others are planets and challenger Is space vehicle.

Words Consistency relationship

Example .,

a) Force b) Cource c) Horse d) alack e) Where

Answer: (b). All words consist of 5 letters. except (b).

FINDING THE ODD LmER GROUP

Example: a) aaC b) BaC c) abC d) BCa e) CBa

Answer: (c). Thls·item has two small letters. All Other items have only one 'small' letter.

There are various relationships which can exist between the items. Several types of relationships have been identified and covered below. You do not need to remember the names of these types. You must understand the relationship and be able to solve the few exercise give with each type.

Relationship Based on position of Letters

Example:

a)AD

b) BE c) CF d) DG e) EF

Answer: re) In each poi'. two immediately following letters have been skipped, i.e. ArBC)D. This relationship is in all pars. except re) in which there is no skipping at all.

Small and Capital Letter Relationship

Example:

a) ABc b) BCd c) CDe d) DEf e) Efh

Answer: (e). Each pai' contains two capital letters and one small letter and natural alphabetic sequence is fallowed. In (e) there is only one capital letter and between 'f' and 'h' alphabetic sequence is not followed as letter 'g' has been skipped.

Vowel-consonant relationship

Example :'a) AbdeE b) IfghO c) ApqrL d) UimnE e) EwxyO

Answer: (c) Each group of letters consists of 2 vowels, one each in the beginnig and end. Between the 2 vowels, three letters in alphabetic sequence are given. In (c), there Is only one vowel.

Repetition and Skipping relationship

Example a} aab dee b) bbc eft c) ffg IjJ

d) IcIcI noo e) Irr ggh

7

Answer: (e). In each group. the initial end final letters are repeated and an intervening letter is omitted. Note followinl1exp!antion:

aab(c)dee

bbc(d)eH

ffg (h) iD

kid (m) noo

Letters in brackets are the one skipped. Letter Form Relationship

Example (a) T b) H c) K d) E e) S

Answer: (e). All letters are formed by straight lines except S which is curved letter.

CODING AND RECODING TESTS

Codes are used for conveying secret messages from one place to another. especially in the defence services. These codes are based on various principles/patterns. which are required to be deciphered by the receiver. so as to get the correct message.

Coding and decoding tests are becoming very popular in competitive examinations. They call for careful observation and anlytical aptitude. These tests can be broadly classified into several categories.

Lettercodlng

The letters of the alphabet may be used in the coding scheme. The letters to be coded are allotted other lellers to stand for them. Questions on lettercoding can be of different types. Examples of the important ones are given below.

ANALOGICAL LETTER CODING

These codes are based on the analogy given in the question itself as shown below.

Form 1

Example: If 'SPTFA' stands for 'BLADE'. how will you code 'BALE'? Answer: 'STPA'

'SLADE' has been coded as 'SPTFA'. You will see that all the letters In the word 'BALE'. Which have to be coded. ore also therein the word 'BLADE'. Hence all that needs to be done is to choose the relevant code letter's from the code word 'SPTFA'. Thus. S becomes S. A becomes T.L becomes P •. and E becomes A. Therefore 'SLADE' will be coded as 'STPA'.

8

Form 2

Example 'XEROX COPY OF Bill WAS SENT' is coded as 'ZGTQZ EQRA QH DKNN YCUUGPV'. Based on this coding scheme, tlcl< the code from the answer choices for each word given in Hie bold letters in the beginning of each question:

1. WATER a) GVCTT b) YCVGT
c) EQARR d) EAVER
2. BUST a)DNKOV b) DNKUQ
c)DNKVU d)DNKUV
3. CLOSE a) ENQUG b) ENQVG
c) EQNUG d) ENOUG
Answers: 1. (b) 2. (d) 3. (a) Explanation

1. 'WATER' look for 'W' in key. You find it in the word 'WAS'. The corresponding code letter is 'Y'. This eliminates all answers except (b).

2. 'BUST' A quick. look at the answers will show that the first three letters, 'DNK' of all the choices are the same. Therefore it will be no use checking the code for 'BU'. Check the code for the fourth letter, 'S'. You will find that it is 'U'. This

eliminates answers (a) and (c). Check the code for fore the answer is (d).

'T'. This is 'V'. There-

3. 'CLOSE' A quick look at the answer choices and you see that it is no use checking the code for the frst letter 'C'. Check the code for the second letter 'l'. This is 'N'. This eliminates answer(c). Check the code for the third letter '0'. This is 'Q'. This eliminates answer (d). Check the code for the fourth letter'S'. This is 'U'. So the correct answer is (a).

The 3 questions In the above example show you that a quick observation of the answer choices tells you which letter should be checked first. If the letter and their sequence, in more than one answer choice, are the same, do not waste time checking ther code. First check the letters which are different. Finding ways like this can save you valuable time.

Form 3

Example: Study the following code and its key. Using this code, decode the codes written In bold letters in the beginning of each question:

Code: l X P Z J Y Q M N B Key: b a e s p r h I g t

1. J Y P Z Z a) grass

c) brass d) gress

2. Q P M N Q B oj cought b) matery

c) fights d) height

b) press

3. Z J Q P Y P a) shpere b) thrown

c) sought d) special

Answers: 1 (b) '(d) 3. (a) CODING WlTI. A SPECIFIC PATTERN

Here, letters are allotted an artificial value but in a specific pattern. You

have to Irst determine the pattern involved and then solve the question.

Example: If 'MAILED' is coded as 'N S J M FE', how will you code the word 'ACTED' ?

Answer: 'B D U F E'

Here each letter stands for the next letter In the sequence. A becomes S, B becomes C,D becomes e, and so on. Therefore 'Acted' becomes'S D U F E' Some commonly used patterns are given in this section.

Form 1: Forward Sequence Pattern

Example: If 'QUIZ' is coded as 'RV JA', how will you code 'CLASS' ?

Answer: 'D M B T T'

Q becomes R, U becomes v. I becomes J, Z becomes A. Here, each letter is coded by the next In alphabetic sequence, with the lost letter of the alphabet 'Z' becoming the first, 'A' . Therefore 'CLASS' will be coded' OM8TT'. You can use the numbered alphabet to help you to recognize the pattern. Write this down and keep it before you while solving such questions.

Form 2: Backward Sequence . Pattern

Example: If 'DATE' is coded as 'WZGV ' ,how will you code 'COME' ?

Answer: 'XLNV'

Here, the coding Is done in baclcward alphabetic order, i.e., A Is coded as Z,B as X. C as W, and so on. Th$ numbered alphabet will help you to identify the pattern In such questions)

10

SERIES COMPLETION

This chapter deals with questions in which series of numbers or alphabetical letters are given. which are generally called as terms of the series. These terms follow a certain pattern throughout. The candidate is required to recognise this pattern and either complete the given series with the most suitable alternative or find the wrong term in the series.

TYPE 1 : NUMBER SERIES Case 1 : Completing the Given Series

Ex.I. Which number would replace question mark in the series 7, 12, 19,1,39. a) 29 b) 28 c) 26 d) 24

Sol. Clearly, the given sequence follows the pattern ; +5,+7,+9 .. i.e.,7+5=L2,12+7=19 ... .'. Missing number = 19 + 9 = 28

Hence, the anser is (b).

Ex.2. Which is the number that comes next in the sequence; 0,6,24,60, 120,210,1

a) 240 b) 290 c) 336 d) 504

Sol. Clearly, the given series is I' - 1,2' - 2, 3' - 3, 4' - 4, 5' - 5, 6' - 6. .'. Next number = 7' - 7 = 343 - 7 = 336.

Hence, the answer is (e).

Ex.3. Which is the number that comes next in the following sequence? 4, 6, 12, 14, 28, 30, ( .... )

a) 32

b) 60 c) 62 d) 64

Sol. The given sequence is a combination of two series; I. 4, 12,28, ( .... ) and II. 6, 14,30.

Now, the pattern followed in each of the above two series is; + 8, + 16, + 32, ....

So, missing number = (28 + 32) = 60. Hence, the anser is (b).

Ex.4.Find out the missing number in the following sequence: 1,3,3,6,7,9,1,12,21.

a) 10 b)11 c) 12 d) 13

Sol. Clearly, the given series sequence is a combination of two series:

I. 1,3,7,?,21 and 11.3,6,9,12

The pattern followed in I is +2, +4, .... and the pattern followed in II is +3. Thus, missing number = 7 + 6 = 13.

Hence, the answer is (d).

1. 46,13,59.15,74,17, " 19

I) 91 2) 843) 93 4) 110

2. 8,35,12,31,18,25,26, __

I ~ 27 2) 19 3) 17 4) 20

3. 23.321,34,431, 45,

I) 652 2) 421 3) 326 4) 541

4. 10,16,13,13,18,8,25, __

1) 7 2) 1 3) 23 4) 3

5. 7.9,16.9. 11, 20, __

I) 23 2) 10 3) \3 4) 11

6.1.9.19,31,45,

I) 49 2) 46,3) '314) 61

7. 36.9.26.30.11, __ , 25.13.12

1)41 2)193)214)18

8. 4. 6. 22. 6. 8. 44, 8, 10, __

I) 74 2) 66 3) 54 4) 88

9. 7.20,12,15.19.8. _._. I

I) 27 ;!) 28 3) 24 4) 11

10. S. 19. 13, 14. 18.9.23.

IIA 2) 27 3) I 4) 9

117.20.9,17.11, .13.11

11\221\33)\44)\5

12.9. \2. 17. .33

1) 22 2) 24 3) 25 4) 30

\3. 8, 24, 13, 19,20, 12, _

1)252)\83)204)29 14.3."5",14,5.7,32.7.9,

I) 511 2) 63 3) 804) 50

15.25.9,16.30.7, __ , 35. 5, 30

I) 23 2) 49 3) 5 4) 15

16.5.7,12,9,11.20. .15.28

1)152)133)194)16 17.1. 9. 25. 49,_

I) 63 2) 81 3) 69 4) 64

18.13,132,26.264.39, _

I) 285 2) 341 3) 396 4) 386

19.12.28.16.24,22,18.30, __ ..

I) 16 2) 14 3) 184) 10

12

I EXERCISE I

20.54,11,65,13,78.15. __ ,17

I) 68 2) 85 3) 90 4) 93

21. 1.8.27,64. __

I) 81 2) 225 3) 1254) 94

22.5.8,13. __ .' 29

I) 18 2) 19 J) 21 4) 20

23.7,21, 12. 16. 19, __ , 28

1) 9 2) 10 3) 23 4)·26

24.4,6,22,6,8.44,8, 10. __

I) 74 2) 60 J) 54 4) 91

25. 15.7.8,20.5, ,25,3,22

1) 25 2) 15 J) 16 4) 12

26.7,9.16,11, 13.24, __ ,17,32 1»)52)163)214)11 27.12,121,24,242. __ .363

1) 26 2) 36 3) 63 4) 60

28.11,29,15,25,21,19,29,

1)14 2)93)114)12

29.65, 11.76.13,87. 15. . 17

1)92 2)943)964)98

30. 1,3.9.27, __

I) 81 2) 243 3) 729 4) 63

31. 3, 10,29,66, _, 218

I) 128 2) 278 3) 127 4) None

32.15,9,24,7,17, ,21,3,18

1)')5 2) 4 3) 21 4) None

33. I, 1,2,3,5, , __

1) 8. 12 2) 8,12 3) 2,4 4) None

34.2.6,4.8,4, 10,

1) 6 2) 8 3) 4 4) 2

35.7,35,8,40,9, , 12,

1) 45,122) 12,603) (5,604) 12, 12 36,1,1,4,,8,.9,27, __

'1) 16 2) 25 3) 12 4) 36

37.3. 1,3,3,9,27, _

I) 243 2) 3 3) 248 4)

38. 19,14,38,28,57,42, .. __

I) 36 2) 723) 764) None

39.7, IS, s, 22,14, __ ,19

I) 31 2) 323) 334) None

40. 19, 39, 79, 159, __

I) 259 2) 319 3) 159 4) 29

41. 11,121, IS. ,19,129

I) 22 2) 165 3) 161 4) 125

42. 11,19,32,50, __ 1)742)733)684)63

43. 67,57. 52.47,37, 32,27,
I) 42 2) 52 3) 67 4) 17
44. 8,10,13, __ ,18,22
I) 17 2) 15 3) 164) 14 45. 13,91,17,21,147, __

1)119 2) (173) 121

46.8,10,12,15,18,22, __ , 31

I) 25 2) 24 3) 26 4) 28

47.11,19,32,50, __

I) 73 2) 54 3) 63 4) 62

48.5,7,15,27,25, __ 1)402)353)474)37 49.97,87,82, __ ,77,67,62,52

I) 63 2) 62 3) 72 4) 65

50. 12, 13,20, ,28,29

1)212)273)264)25

.51. 8, 10, 13, ,18,22

I) 17 2) 14 3) 15 4) 16

52.7,35,175, __

I) 875 2) 245 3) 675 4) 625

53.6, 18,54,162, __

I) 71 2) 41 3) 54 4)486

54. 11,13,17,25, __

I) 33 2) 35 3) 42 4) 41

55.67,57,52,42,37,27,22, __

I) 12 2) 7 3) 5 4) 15

56. 1,2,5,26, __

I) 209 2) 65 3) 626 4) 677 57.3,21,6,42, __ , 63

1),12 2) 9 3) 7 4) 6

58.96,6,48,18,24,54, __

I) 28 2) 8 3) 12 4) 27

4) 104

59.6,10,16,26.42, __

I) 56 2) 58 3) 78 4) 68

60.2,5,10,17, _

I) 24 2) 26 3) 25 4) 27

61.15,17,19,22, . 28

I) 25 2) 30 3) 32 4) 24

62.2,8,9,17,18, __ ,29

1) 26 2) 25 3) 28 4) 24

63. 1~2, I~8, 20,14,25

I) 18 2) 16 3) 19 4) 20

64.12,16,20,21,27, .. 34

I) 33 '2) 303) 31 4) 28 65.7,8,7,9,8, __ , 11 1)92)103)114)12 66.6,18.9,27, __ , 36

I) 9 2) 183) 194)12

67.5,1, 10,7. 15, 13,20, 1)172)183)194)25

68. 1,5,6.11,17,

I) 28 2) 27 3) 25 4) 23

69.6, 13,20. .34

1)272)253)214)19

70. 5, 12. 15.24,25, __

I) 42 2) 36 3) 35 4) 30

71. 5. 10, 17.26, 1)362)373)424)49

72.11, 13, 15, 18, 21, ,29

I) 20 2) 23 3) 25 4) 27

73.64,4,32,8, 16, 16, __

I) s 2) 16 3) 24 4) 32

74.8, II, 16,23,32,

I) 38 2) 48 3) 41 4) 43

75.12,13,18,19,24,25,

I) 26 2) 32 3) 30 4) 39

76. 8, 56, 392, __

I) 2744 2) 253 3) 1762 4) 7879

77. 6, 8, 12,20, __

I) 28 2) 24 3) 36 4) 32

78.8,9, 13, 15, 18, __

I) 20 2)21 3) 244) 30

13

79. 7. "'. t. 3. 9, .16,4, 11, 55,_~
1) 44 2) 33 3) 50 4) 5
~O. 23.33, 34, 44. 45. 55.
I) 56 2) 46 3) 54 4) 63
81. 9. 17. ~O, 48.
I) 52 2) 6() 3 ) 71 4) 61
X2 14, 154, io. 176, 18,
Il 19X 2) 242 :. ) 186 41 \:;C;
83. 12. D. 20. lB.29
II :' I :'1 "'.7 3 ) 26 4 ) 25
84. R. IG. 13. 18.22
1) 17 21 14 3 ) 11 4l I k-
X5 7. 35. 17).
II 875 2) 245 'I 6i5 41 (-.];':
X6. (,. IX. I R. 54. 162
II 71 2l 4 I 31 54 4) )4
XI. S. 10. 12. I', 18.22. .' ,1
II ::5 cl ]4 3) 26 41 2:-:
XX I I. I o. 32. 50,
II 73 2) 54 .lj ()1 4) 62
x». 5.7. 15. 27. 25.
II 40 2) .15 :1 ) 47 41 37
90 7. 21. 2L (i3. 1 X9
II 77 21 75 3', 69 4 ) 63
'II 9. 11. 15.23,
II 29 2) JI 3) 39 4) 35
92 9.45.225. I) 13.,9 2) 625 31825 4) 1125

93.14.15.21. __ .--,,30,3\ 1)232)263)254)2'1

94.5,7, 15,27,25. __

I) 35 2) 23 3) 47 4) 37

95.2,7,9,3,8, 11,4,9,13, _._, 10, 15

1)6 2)53)114)12

96 1., 6, 11. ~~_, 27, 33 1)182)173)164)13 97.6,24, If' _~, 18,8,24

I) 48 2) 1(3) 284)6

98.1, 2, 2, 6, 3,12,4,_,5,30

1) 6 2) 5 3) 16 4) 20

14

99. 15,24,35,

,63,80

118 2) 42 31 4941 None

100.97.96,92.83,07.42, 1)362)1(»)244)6 101.19,)<),79,159, __

1',2511 2) )1~ 31219 1()2.!9. 14, lR, 2(;. 57. 42.

4) NOlle

11 5(l :) M:' ]) 764) None

[01.2.;,). S. 4. 15,.5.24,6,

I) ~~ ~) 2X T) 12 4) 3~,

104.49,61.51, ()S. 5l. (,7. 55 69.57.

1) ()7 2) 5()::;) 714) /_l

I05.!2(lA:?16X. 2~,2. 7~. ~I)(L Jl~L ro«

1') ()(l 2) 77,) 71) -tJ 72

J07_Ru_ ·10. . --14. ()(j. 4~. 9~. )2. I

II S.'

IOX.4. 25.121 2N!).5»,

1) 576 2) ()25 3) h7!) 4) ~41 _:.;) l)()\

\IJ(). L ::. 0, 50.

1') ,)50 2} ~q .Il ~()n 4) ~()~ 5) 3hO

110.1. 20.126.

I) 1200 2) 1208 11 1220 5) J 01 (,

111.1.4. 21l. 1(,8.

I) 374 2) 1374 .1) 2)74 41.;"/1'1

"1} 1110

112.G, 7,16.51, 20R.

I) 1020 2) 11130 )1 I04() 4) 1045 5) 1055 113.5,4,6, 15. 56, 275,

1) 5100 2) 5102 3) 1618

4) 1642 5) 1644 114.3,4,15,80,567,

I) 5100 2) 5102 3)5111

4) 5112 5) 5224

115.100, 110,210, 22t, 342, 354,

I) 396 2) 4<)8 3) 412 4-j 444

116. I, 10. 6(J, 469.
I) ]7()O 2) 3725 1) 1740 4) }745
117. 18,37,75, 15 L 303, 607,
I) 1234 2) 1432 ] ) 1215 4) 1225
118, 121. 169.289,361. :'129.
I) 625 2) 676 3) 729 4) 784 5) 9UO
119. 5,3D, 130.6,42,222, 7, 66.
I) 30 2) 68 3) 130 4) 222 5) 350
12(). 2, 17, 101,362,
I) 'JOO 2) 902 }) 9h2 4) 1001 5) 2002
121 20,4, 80, 5. :H), 6, 18O, 5, 70,5, }50,
I) 10 2) II 1 ) 12 4) 1:1 5) 14
122. 2,28, 126, 344, 730,
I) 1332 2) 1001 1) '''Li 4) 217 5) NOIlL"
123. 132, 182, 306, }80, 552,
I) 6(JO 2 ) 676 3 ) 702 4) 870 5 ) 902
124. l:'i. 31.95.239. • 8'-)'::;. 1471
I) 141 2 ) 3Rt) .:; ) 423 4) 495 5) 505
125, 10,35. 135, 36(), 11R5,22gS
I) 390 21 43() 3 ) 540 4) 650 5) 760
126. 5. 12,32.9L267,
1) 704 2) 604 3 ) 504 4) 404 5) 7~4
127. 5,6, 14,45, 184,925,
I) 5556 2) 1489 3) I(I()() 4) 990 5) 955
128. 4. 3. 4. 9, 32,
I) 155 2) 150 ::; ) 110 4) 75 5) 55
129. 10,9. 16.45. 176.
I) 275 2) 475 }) 675 4) 875 5) 1025
130 2,7.23, , 22(), 665
I) 4'l 2) 58 3) 24 4) 72 5) 129
131. 5.9, 14, 19,22, 19,
I) 18 2) 16 3) 12 4) (, 5) 2
132. I, 2, 8, 33, 148,
I) 250 2) 325 3) 485 4) 575 5) 765 15

a) B b) 0 c) F d) H
a) 1 b) 3 c) 2 d) 4
12
~ BEl
6

a) 45 b) 75 c) 105 d) 225 a) 25 d) 121
13
a) 6 bl 5 c) 8 d) 9
® ® ®
a) 10 b) 11 c) 12 d) 14
14. 6
14

a) IJ9 b) JL24 C) JK18 d) JL9 a) 11
b) 13 c) 15 d) 17
15. 15 13
21 18
a) 151R b) 151P c) 20lR d) 20lP 10 26
I ~ I L I ; I : I ~211~ I a) 14 b) 15 c) 16 d) 17
16.
a) HI12 b) HIS 0) RI12 d) RIS

18 MISSING NUMBERS

DIRECTIONS; Find out the missing numbers:

a) bl 5

c) d)

a) 94

b) 86

c)82

d) 78

._.
8 3 4
7 6 1
2 4 ?
~._ 1-- 4
13 5 a) L' b) L' c) L" d) L'
10 0 T M
C
G N P
a) B b) F cj G d) H
11 a) P and 8
c) Sand 8
17
a) F and 21
ciGand21
18
a) 33
b) 35
c) 37
d) 39
19 §--1
a) J!!I b) .r- biPand10 dl Sand 10

b) F and 24 dl G and 24

S'

[D"!

d) L"

20

a) S)

21

E"

c) L Ii.!

b) s-

c} DJ

a) lI2 b) MI2 c) MI4 d) NI2
22 @
a) 54
b) 56
c) 58
d) 60
0
23
a) 40
b) 41
c) 42
dl 43 24

a) E

b) F

c) G d] H

d) D~

25

~
6 .
a) 1 b) 2 C) 3 o) 4 DIRECTIONS 26 TO 30 :

The following questions are based on artificial values Find the correct relation

26 1149 x ~4" 676 65 x 59" 625 then 99 x 95"'

a) 1024 rt 1124 ct 1224 d) 1324

27 il 123 x 234 "753 345 x 456 " 108 then 968

x 869 :;: ?

a) 1837 b) 8371

CI 1738 dl 7318

28

If 25 x 55 x 65 = 435 24 x 44 x 84 :::: 456 then 66 x 76 x 86 = ?

a) 864 bl 466 c) 684 o) 846

29

if 346 x 542 = 222.345 + 455 ~ 200 then 355

x 245 ::;. '(

a) 600 bl 500

c) 200 dl 150

30 If 49 x 96 = 82,55 x 86 = 42. then 99 x 97::; ?

a) 34 b) 43 c) 44 dl 46

RULES

A = Add the square to its cube

B = Add thrice the number to Its cube C = Subtract the square from its cube

o = Subtract thrice the number from its cube E = Multiply the number by eight and subtract

double the number With you have multiplied

QUESTIONS:

31 12,36, 150, 392, 1452

a) A b) S c) C

c) D e) E

32 4, 14, 36 76, 140
a) A b) B c) C
dl D el E
33, 4, 18,48, 100, 180
a) A b) B CI C
d) D e) E
34. 18,52, 110,198,322
a) A b) B c) C
d) D e) E
35. 40,80,120, 160, 200
0) A b) B c) C
d) 0 e) E 17

NUMBER ANALOGY

lhi.s section deals with two types ofquestions .

Choosing a similarly related pair as the given number pair on the basis of the relation between the numbers in each pair

II. Choosing a number similar to a group ofnumbcts on the b~\s:s otccrtuin common properties that they possess.

IILLUSTRA TlVE EXAMI'UcS I

Ex.!.) : I I : : 7 : .)

a) 22

b) 29 c ) 18 til 51

Sol. Clearly. 3' + 2 = 11 Now'. r j-:2 - 51

So. if the first number is x , the second number is x +~. rhus, the relationship is x: x' + 2.

Hence. the answer is (d}.

Ex.2.324; 162

a) 64 : ~6

h) 2· I

c) n: III

til IH' 112

Sol. Clearly. the relationship is 2x' x. lhi s relationship exists in (h)

So, thc correct answer is (b).

Ex.3. Which number is like the given set or numbers 'j Given Sd : (J, J 7. 31)

a) 5

h) IS

c) 45

d) 4')

Sol. Clearly the number in the given set arc all prime numbers. 5 is ,dsu:t primc number and so he longs to the same group.

lienee. the aIlSV"Tr is (a).

Ex.4. Which set of numbers is like the giveu set '!

Given sct : (48, 24. I::!)

0) 44.22.10 h) 46.22. II c) 4f1. zo. 10 el) 42. 2(), 10

Sol. Clearly. in the given set. the first 1l111l1bn us twice the second and the second number is twice the third. A similar relationship exists between.the numbers in 11k group (40. 20.10).

Hence. the answer is (c).

18

12 42:'

1. () 20:' 1132

1) 42 2) 110 1) 210

: 13.12

I) 420 2) 341 :;) 277..

3. () '.~:. : 9:Hl

I I 72

4. h 42:

I I I II

S. 2072:' I) 15() (1. 12 (J():

7..) iJ2 .1) I! ()

f)."W

~) 2{) .1) :1(4) 4() I (j~h

2) llO J) 272

1J.,fJ

4) 272

4) 21 ()

41 1.12

4) 342

1) ')0 2) I I () .,) 132

7 .. ..10 130 134:

1\ 130 21 2~2 3) J:,() 41 520

1) III 2) 35(j Jl 520

lJ .. () 12:' : :-:7U

I) 7S() ~I 70.' I)';"!

1 () ()1() . ()')2

1) ~52 2) 6:'0 II 75h

11.. 1.1~ .106:'

11 lXO 12.6 : 24 : ..

I I 120

1) )')1 )) 7':06 1320

2) 210 3) 3]6

13.24.120:' :U20

11210 2) 504 )) 720

14.14:36:' :1364

I) 234 2) 364 3) 536

1536:140:: :1164

II 364 2) 536 3) 1030

16.36'140:"

II 536 17.12: 36 I) 150 18.12: 36::

I) 150 19,4: 48 I) 100

, 1364

2) 756 3) 1030 1452

2) 252 3) 576 1452

2) 252 3) 392

900

2) 180 3) 294

41 156

4) 73X

4) X70

-\) X70

4) 504

4) 5'iS

4) 1030

4) None

4)None

4) 1100

4) 576

4) 648

I EXERCISE

20.4 48 I) 21.4 18·

.900 :nI(31124)14 121)k

I) 100 2) 180 J) ·2~)..j

2211R 12l)R

I) 198 2) 488 .'1 7(1'

23 I X 11 () :. , I?l 0

1)'222)488 24 I H I no .:

n 9110 2) (,48

25.1X ~6:

3) 707..

1'10

,') 448

J) IUO 2) 1XO 3) 294

co.IX:IX :IXO

1) IXO ~) 29,+ 3)'.2:

2XI4:P\t): 661

1) 2R ~l 26 ~) 2-+

:2() .. ~ S : M':

1) (iO 7.. ) ()2 0\) 1>-1

'D. 20 ' 6X

I) 56 ~ 1 12 «o

I) 110

:,2,4 : 1..2

I) 20

33.2 : 14

I) 52

~4, 2 : 2

1) 42 ~5, 2 : 6 .:

I) 10

36. H : 68 .:

I) 12 37.37:217 I) 82 38.18:68::

I) 144

: 5211

2) 7:! 1) ()()

720

::) 9() .1) 7~' XII

2) 24 .1) 3h 76

2) ,IX J) 1 I

1) 49 .1) S() 60

2) 20 .'J]o 1511

2) 14 3) 16 730

2) 90 3) I)() 588

2) 150 3) 252

I) l)7U

41 I XII

·1)

-1") )1)

-1) 4X

41 Iii

·il

·il

4) 73R

19

39.33: 138:: : 770
I) 189 2) 248 3) 315 4) 390
40.4: 68 :: : 1342
1 ) 10 2) II 3) 12 4) 13
41.6: 210:: no
I) 7 2) 8 3) 9 4) 10
42.6: 10:: 1342
I) 110 2) 90 3) 132 4) 210
43.30:210:: : 990
I) 130 2) 222 3) 350 4) 520
44.100: ISO:: : 110;)
I) 1100 2) 900 3) 810 4) 548
45.110: 140:: : 1030
I) 1030 2) 1000 3) 970 4) 648
46.306: 380 :: : 870
1) 992 2) 756 3) 650 4) 400
47.56:182:: : 992
1 ) 650 2) 676 3) 756 4) 870
48.30:132 : 380
I) 306 2) 240 3) 182 4) 132
49.12:156 : 506
1 ) 22 2) 24 3) 26 4) 28
50.16: 240:: : 600
J) 24 2) 25 3) 264) 28 20

TYPE : CHO(1)II~t> THE ODD NUMERAL

In this type of questions, certain umbers are given, out ofwhich all except one arc alike in some manner while one is different and this number is to be chosen as the

answer.

!ILLUSTRATIVE EXAMPLES I

Directions: ChOO.H! th e n umber which is different from others in the group.

Ex.I.a) 12 b) 25 c) 37 d) 49 e) 57

Sol. 37 is the only prime numb4er. Hence, the answer is (c).

Ex.2.a) 8 b) 64 c) 125 d) 216 e) 28

Sol. Each of the numbers except 28, is a perfect cube. Hence, the answer is (e).

Ex.3.a) 2 I b) 36 c) 49 d) 56 e) 91

Sol. Each of the numbers except 36, is divisible by 7. Hence, the answer is (b),

Ex.4.a) 751 b) 734 c) 981 d) 853 c) 532

Sol. In each number except 751, the difference of the third and the first digit is the middle one. Hence, the answer is (a).

Ex.5.a) 381 b) 552 c) 729 d) 903

c) 295

Sol. Each number except 552, is an odd number. Hence. the answer is (b).

Ex.6.a) 8314 b) 2709 c) 1315 d) 2518 e) 3249

Sol. In all numbers except 8314.the sum of first three digits is equaJ to the unit's digit. Hence. the answer is (a).

Ex.7.a) 48

b) 12 c) 36 d) 24 e) 59

Sol. In all numbers except 59. the unit's digit is twice the ten's digit.

Hence. the answer is (e).

Case II : Finding the Wrong Term in the Given Series Ex.l.Find the wrong number in the series: 7.28.63.124.215.342.511

b) 28 c) 124 d) 215

a) 7

e) 342

Sol. Clearly, the correct sequence is

2) - I, Y - I. 43 - I, S·\ - I. 63 - I, 7.1 - J, g-' - 1.

28 is wrong and should be replaced by (3' - I) i.e. 26.

Hence. the answer is (b).

Ex.2.Find the wrong number in the series: 3.8, 15.24.34.48.63

b) 24 c) 34 d) 48 e) 63

a) 15

Sol, The difference between consecutive terms of the given series are respcctivery 5.7.9.11 and 13.

Clearly. 34 is is a wrong number and must be replaces by (24 + I I) i .e. 35. Hence. the answer is (c).

DIRECTION (1 TO 25) 13. a) 30 b) 56 e) 132 dl182 e) None
Four of the following five are alike in one-
way or the other. Find the one which does
not belong to the group.
a) 2 b) 3 e) 5 d) 7 e) None 14 a) 30 b) 130 e) 350 d) 1342 e) None
at 3 b) 5 c: 7 d) 11 e) None 15 a) 252 b) 420 e) 600 d) 702 e) None
a) 11 b) 13 0)17 0) 19 e) None 16 a) 930 b) 1056 e)1190 dl1342 e) None
a) 11 bl 31 0141 d16~ e) None 17 al 130 b) 350 GI738 d) 1342 e) None
a) 99 b) 89 G) 79 dl59 e) None 18 a) 1984 bl 1974 c)1964 d) 1944 e) None
81 71 bl 73 C) 79 dlS3 e) None 19 aj 10 bj 68 GJ 222 dl520 e) 1010
a) i3 hi 83 c) 93 d) 103 e) None 20 a) 18 b) 39 crse dj 105 ej 150
a) 121 b) 169 GI225 dl289 e) None 21 a) 17 bl 30 ci se dl90 e) 132
0) 121 t» 529 c) 361 d) 841 e) None 22 a) 9 b) 25 c) 49 dl64 e) 144
10 a) 576 bj 676 el784 dl900 e) None 24 aj 900 bj 488 c) 180 d) 48 e) 4
11 a) 729 b) 512 c) 343 d)216 e) None 25 a) 970 b) 448 e) 198 d) 52 e) 2
12 0) 729 b) 676 e)625 d) 576 e) None 22

LETTER SERIES

This type of-questions usually consist ora sereis of small letter which Iollow n certainpattern. However, some letters are missing from the series. These missing letters ae then given in a proper sequence as one of the alternatives. The candidate is required to choose this alternative as the answer.

Example: aab _ aaa _ bba __

a) baa h) abb c) bubd) aabe) bbb

Sol. We proceed step by step as shown below:

1. The first blank space should be filled in by 'b' sot hat we have two a's followed by two bs,

2. The second blank space should be filled in either by 'a' sot hat we have four a's followed by two bts. or "b' so that we have three a's followed by three bs.

J. The lase space must be filled in by 'a'.

4. Thus, we have two possible answers: 'baa' and "bba ' But only 'haa appears in the alternatives. So, the answer is (a).

5. In case, we had both the possible answers in the alternatives, we would have chosenthe one that forma more prominent pattern. which is aabbl aaabbb I aa. Thus, our answer would have been "bba'.

Correspondence Series- e- This typc of series consists of three sequences with three different clements (usually ccpl ital leuer. digits and small letters). On the basis of the similarity in positions in the three sequences, a capital letter is found to correspond with a unique digit and a unique small letter. whenever it occurs. The clements to be filled in at the desired places,

Consider the following example:

Ex. In the following series, choose the alternative which contains the nucmralsa to be filled in the marked spaces, in the correct order:

13

D

4 I 3 2 b c

CABDACI3

? ? ?

a) [,2,3,4

b) 2,3, 1,4 e) 1,2,4, 3 d) 2, I, 4, 3

Sol. Clearly. in the second series, 1 occurs at the same position as D occurs in the first series. So, 1 corresponds to D. Thus, the first question mark below D is to be replaced by I.

Now, in the third series, c at the eight place corresponds to A in the first series, while c at the sixth place corresponds to 2 in the second series. So, 2 corresponds to A. Thus, the second question mark below A is to be replaced by 2, In the third series," at the first place corresponds to B in the first series and a at the third place corresponds to 4 in the second series. So. 4 corresponds to B. Thus, the question mark below B is to h~' replaced by 4,

23

Now, only 3 remains, So, 3 corresponds to C. Thus, the question mark below C is to be replaced by 3, Thus. DACB corresponds to 1,2.3.4.

Bence. the answere is (a).

TYPE: CHOOSING THE ODD LETTER GROUP

In this type of questions, usually five groups of letters are given, Four kofthern me similar to each other in some manner while one is different and this is to be chosen by the candidate as the anser.

r---------------,

I ILLUSTRATIVE EXAMPLES I

Dtr-ec rions : Choose the group of letters wh ich is different from others.

Ex.l.(a) BD (b) IK(e) PN

(d) SlJ

(e) WY

Sol. Clearly. the answer is (c), All other groups consist of

two alternate letters in order, while in this group they arcin reverse order.

Ex.2.(a) BCD (b) KMN (e) QRS (d) Gill (e) WXY

Sol. Clearly. the anscr is (h). All other groups consist of three consecuti ve letters while this one doesn't.

E x. 3.(a) POCO

(b) KLiZ

(e) BlIDX (d) FQMV (e) ARTG

Sol. Clearly, the answer is (d). All other groups consist of one vowel each but this group doesn't contain any vowel.

Ex.4.(a) CZHK

(b) MLi\G (e) XUBU (d) SI':NO (e) YDFP

Sol. Clearly, the answer is (c). This is the only group in which one letter has been repeated.

Ex.5. (a) B DO K

(b) JLOS

(e) NPSW (d) MORLJ (e) HJMQ

Sol. Clearly, the answer is (d). I n all other groups, there is a gap of I letter between first and second letters, 2 letters between second and third letters, and] letters between third and fourth letters.

E x, 6.(a) CnL (b) PSVX (e) KMPS (d) ORUX (e) QTWZ

Sol. Clearly. the answer is (b). In all other groups, each letter moves 3 steps forward to obtain the next letter.

Ex.7.(a) DkUZ(b) LPuB

(e) FoMY (d) UXeN (e) WaQS

Sol. Clearly, the anser is (a). In all other groups, the smaller letter is a vowel.

E x, 8.(a) FCODE (b) TRQPS (c) KJIIMF (d) KHGJI (e) XVYZW

Sol. Clearly, the answer is (c). All other groups consist ofconsccutivc letters though not in order.

Ex.9.(a) AUgPZ

(b) MXiDV (e) KFeCO (d) YGLhT (e) UHmQY

Sol. Clearly. the answer is (d). In all other groups. the smaller letter is the middle one. E x, lO. (a) DXCLQZ (b) PFZUBM (e) XGKNTY (d) NWMBHJ (e) GJMQVX Sol. Clearly. the answer is (b). This is the only group containing a vowel.

24

DIR~CTlONS : 1 TO 25

The following questions are based on Letter Series. In each of the following questions find the missing letters.

a) aba

ab _ abaabaa _ aabaab

b) aab

c) abb d) aaa

a) z z z

zy _ zyzzyzz _ zzyzzy _

c) vvv dl zzy

b) zyz

pq _ pqppqpp _ ppqppq

a) ppp

b) qqq

a) abb

aa _ bbaaab _ aa bb

b) aab

Zl _ yyzzzy _ zZ _ yy

a) zyz

b) zyy

a) PPP

pp _ qqpppq _ pp _ qq

b) pqp

c) pqp d) ppq

C) aaa d) abe

c ) zzy d) yyy

c) ppq d) pqp

a _ aaabaaa _ aaa aa

at aaa

b) bbb

c) aab c) abb

Z _ ZlZYZZZ _ zzz _ lZ

a) z az

b) zzy

c) yyy d) xyz

p _ pppqppp _ ppp - pp

al ppp

bl qqq

10 b _ aaabb aab _ aaa

a) bab

b) baa

11 Y _ ZZZ'IY _ lZy _ ZZZ

0) yZY

b) yyz

12 q _ pppqq _ ppq _ ppp

13.

a) qpq

b) qpp

bba ba

cl ppq dl pqc

c) bbb dl bba

c) yzz d) yyy

c) qqp d) qqq

"I aabbbb b) aabbab c) aabbba dl aaaabb

aa ab _ a

14 _ yyz _ yz __ zz _ zy _ z

a) zzzyyy b) zZ1YYY c) zzyyzz dl zyyzzy

15 _qqp_qp __ pp_pq_p

a) pppqqq c) qqpppp

b) qqqppp d) ppqqqq

16 b a bbb aa b ba

a) babbba c) babbbb

b) bababa d) babaab

17 Y _ z _ yyy _ zz _ y _ yz _

a) yzyyyz bl y,yzyz CI yzyyyy dl yzzyyz

18 q _ P _ qqq _ pp _ q _ qp_

01 ppqqpp c) qpqpqp

b) qpqqqp d) qqppqq

19 ab x aaba na baxna _ baxna

a) anxaa C) anaxa

b) anxab d) anaxa

20 _ yzzzyz _ zz _ yzyzzz _

s) zzyy b) zyzy c) zyyz d) zzzz

21 _ qpppqp _ pp _ qpqppp _

a) ppqq b) qqpp c) pqpq d) pqqp

22

aba _ a aaa _ aa

a) aabb b) aaba c) aaab d) abbb

23 _ zyz _ z _ zzz _ zz

a) zyzy b) llZZ c) zyyz d) zzyy

24 _ pqp _ P _ ppp _ pp

a) pqqp bl pppq c) ppqq d) pppp

25.

bam _ amb _ m a bam

al mbabm b) mbaba c) mbaam d) mbama

25

TYPE: ALPHABET SERIES Ex.J, What terms will fill the blank spaces?

Z,X,V,T,R, ( .... ), ( ..... j

(a) O,K (b) N, M (C) K, S

(e) P, N

(d) M, N

Sol. Clearly. the given series consists of alternate letters in a reverse order. So, the missing terms would he P and N_ Hence, the answer is (e).

Ex.2. Which terms comes next in the sequence: nd iy dt yo tj?

(a) mp (b) nq(c) or (d) oc Ie) me

Sol. Clearly. the first and second Ictcrs of each term aTC moved five steps backward to obtain the corresponding letters of the next terms. Hence, the answer is (d).

Ex.3. What will be the next term in: BDF. CFI. DHL.?

(a) CJM (b) WIM (e) EJO (d) EMl

Sol. Clearly, the first, second and third letters of each term arc respectively moved one. two and three steps forward to obtain the corresponding letters of the next term, So. the missing. term is EJO. lienee. the answer is (c).

Ex.4. Which term comes next in the series: YEI3. WeD. UHG, SKI?

(a) <)01. (b) QGL

(c) TOL

(ell QNL

Sol. Clearly. the first letter of each term is moved two steps backward to obtain the first letter kof the next term. So, the first letter 0 the missing term will be Q. The second letter of the first, second, third. fourth terms arc respectively moved one, two, three and four steps forward to obtain the corresponding letter kof'thc subsequent term. So. the second letter in the missing term will be O.

The third letter is alternately moved two and three steps forward to obtain the corresponding letter of the subsequent term. So. the third letter in the missing term will be L. Thus, the missing term is QOL. lIence, the answer is (a)

E.x.5. Which term will replace the question mark in the series:

111m, DGK. HMS. MTB, SBL.?

(a) ZKW (b) ZKU (e) ZAB (d) XKW

Sol. Clearly. the first letters ofthe first, second. third, fourth and fifth terms arc moved three. four. five. six and seven steps forward respectively to obtain the first letter of the successive terms. The second letters of the first. second. third, fourth and fifth terms are moved five. six, seven. eight and nine steps forward respectively to obtain the second letter of the successive terms. The third letters of the first. second, third, fourth and fifth terms are moved seven. eight. nine, ten and eleven steps forward ddddrespectively to obtain the third letter of the successive terms.

Thus. the missing term would be ZKW. Hence. the answer is (a).

Ex.6. Choose the term which will continue the following series: P3C. R5F, T8I, VI2L.?

(a) YI70(b) Xl7M (c) Xl70 (d) XI6 0

Sol. Clearly. the first letters of the terms are alternate. The sequence followed by the numbers is + 2. +3, +4, .... The last letter of each term is three steps ahead of the last letter of the preceding term. Thus. the next term would be X 17 O.

Hence. the answe is (c).

26

PIRECTIONS : 1 TO 25

The following questions are based on Letter Series. In each of the following questions lind the missing letters.

ABCDE KI,MNO 81 JKLMN

cj IJKLM

el HIJKL

lABCD, ?, YlABC b) KLMNO

d) MNOPQ

ABCDE. IJKLM TUVWX. ?,I-ZABC

81 HIJKL III GfJl>JK

01 FGHIJ d) IJKLM

e) JKLMN

ABCDE IJKLM TUVWX, BCoFF?

al KLMNO b) LMNOP

c) MNOPO d) MNOPR

EGVUC. VXEFR rHUlO. ? GrrSE TVGHP

al UQVEC; b) WGHLQ

c) UWFGQ d) fGQVW

e) GHUVVV

zyym A8VUM ? BCUTL. XWAST COTSK

a) YXZAS b) ZYABT

C) XWYZ'r<: d) YXZAB

e) ZXYBS

YBGN, RUZG NOVe MPUB ')

at oove OORS

ci ORWll d) OSXE

OSWA FJNR I,FKP CHMR?

a) IOUZ bl IQUB

c) IOUC d: IOUA

9. LOVA DINS XCHM TYDI ?

ar RWBG bj RXCF

c) RWCF d) RWAB

LMNO. ZASC, PQRS, HIJK ?

a) BCDE bl ABCo

c) YZAB d) XYZA

10 YlWX, UVST, OROP ?

a) MNKl b) KLMN

c) IJGH d) ABYl

ADS, EHF, ILJ, ?

a) PMN b) MPN c) NOO d) LOM

CFED, GJIH, KN ML ?

a) MPON b) POOR

cl OROP d) NOPO

13

ABAC, BCBD, CDCE ?

a) OEFG b) DEDG C) DEDH dj DEDF

DFNr TDFN NTDF ?

14,

a) FNTD C) FNCS

bl FNDT o) FNSS

15

LMN, RST, a) TUV

c) UVW

e) UTV

ABC MNO. BCD b) STU J) RST

16

YlA, RST a) XYZ

c) lAB

e) ABC

GHI, RST, ?

bj YZA dl WXY

17

MNOP STUV ABeD, ? WXYZ

a) IJKL b) JKLM

c) KLMN d) LMNO

18

YZAB GHIJ MNOP UVVVX ?

a} YZAB b) ZASC C) SCDE dl ABCD

19

XAFM VYDK, TWBI, RL;ZG o

a) PSXE bl RQWD c) RUZG dl PSXD

20

LQV CHM. 7

bl RWS di RW!

XCH. SXC a) ORW

c) RWA

e) ORB

21

MNOP. NMLK a] UTSS

c) UTSK

e) ROPO

FGHI UTSR, IJKL ? b) UTSP

d) UTSR

22 ONML, GFED UTSR ?, K,IIH

a) EDBC b) EDBB

c) EDBA d) EDCS

23 BCDE, HIJK, PQRS ?, ,lMNO

a) lABC b) ZASD

c) lABA d) ZASE

24 lASC. lMNO, XYlA, ?, VWXY

a] JKLM b) JKLO

c) JKlN d) JKNO

25 LMNOP, TUVWX, EFGHI, ?, JKlMN

a) STUVW b) RSTUV

c) RSTUW d) RSTUY

27

DIRECTIONS: 1 TO 25 ? SXCH, CHMR
The following question, are based on 14. SXCH, NSXC,
Letter Series. In each C)f the following questions b) MRWC
find the missing letters. a) MRWB
C) NSXC d) NSXB
CDE, HIJ OPQ, XYZ,?
15 STUV, XYZA, EFGH, NOPO, YZAB, ?
a) IJK b) JKl c) KLM 0) LMN
a) KLMN b) JKLM C) LMNO d) MNOP
COE, IJK, RST, OFF 7
16 XYZ. QRS 7,ORS, XY,Z
a) RSK b) STU c) GRS d) PQR
a) FGH b) EFG c) GHI d) FGH
COE. IJK ? XYZ GHI
17 QRS, XYZ. 7, XYZ, QRS
a) OPQ b) ORS c) RST d) STU
a) IJK b) FGH c) GHI d) JKL
OP~, WXY 7, RST EFG
18 ABCD, MNOP. YZAB, ?, WXYZ
a) HIJ bl IJK c) JKl d) KLM
a) IJKL b) JKLM c) KLMN d) LMNO
MNO, VWX. ? TUV, IJK
19 CDEF, IJKL. ORST ? MNOP
a) IJK b I JKL c) KLM d) LMN
a) ZABC b) YZAB c) XYZA d) ABCD
OEFG LMNO. XYZA FGHI ?
20 NOPQ, TUVW, BCDE, 7, XYZA
a) OPOR b) PQRS c) ORST d) RSTU
a) KLMN b) LMNO c) MNOP d) NOPO
BDGK. PRUY, FHKO XZCG?
21 PTXB, GKOS, BGLQ, DINS"
at RTWy b:1 RTWA c) RTWB d) RTWX
a) IOUA b) IOUB
YBGP VYDM, XAFU 7WlEN C) JPVB d) UPVA
a) VYDM b) YENP c) UXCL d) UXCM 22 KPUZ, CHMR, WBGL, SXCH,?
NRYI. PTAK. XBIS 7 HL8C a) RWBG b) WBGK
0) TYOI d) OVAF
a) ZDKU b) ZDKL c) ZOKT d) ZDKV
23 ABCD, IJKL, QRST, YZAB ?
10 ZABC. FGI1I. NOPQ ZASe ?
a) GHIJ b) HIJK
a) TUVW b) UVWX c) VWXY d) WXYZ c) FGHI d) EFGH
11 OPQR, UVWX, CDEF. 7,IJKL 24 ABCD, IJKL, UVWX, CDEF, ?
a) OPOR b) PQRS c) NOPO d) MNOP a) GHIJ b) OPQR
c) NOPQ d) OPOS
12 UVWX, COEF, OPQR, IJKL?
a) PQRS b) QRST c) RSTU d) STUV 25, ABCD, GHIJ, NOPQ, WXYZ, HIJK, ?
13 DGLO. JMRU, KNSV GJOR, ? JMRU al STUV b) UVWX
a) YBGJ b) XAFI cl WXYZ d) XYZA
c) YBGK d) XAFJ 20

TVPE : ALPHABET ANALOGV

In thj s-t ype ofquestions. two groups ofalphabets related to each other ill same way" arc given. The candidate is required to l1nd out this relationship and choose a !-:.rollp o fu lphubet s which is related in the same way to a third group provided in the qucsnon.

ELI.ABCD:OPQR WXYZ:'

(a) H'"GH (b) KLMN (e) QKSr (-I) sruv

Sol. Clearl y, each letter of the lirst group is moved fourteen steps forward to obtain the corresponding: letter ottbc second group. A similar relationship will exist -hctwccn the third aqnd fourth groups.

Hence. the answer is (h).

Ex.2.<)I(lK . MMKO: Y J\ WC : ?

(a) ~U!'(j

(b) VilES

(c l USGA (d)tJESG

S .. l. The First and third Ieuerss of the first group are cactl moved [our steps backward to obtain the corresponding letters ofthe second group, The second and fourth letters ofthe first group arc each moved Iour steps forward to obtain the corresponding letters ofthe second group,

A similar relationship will exist between the thrid and the fourth groups. llcncc , the answer is (d}

I EXERCISE I

Dil·cdiul1.s: In cuch of' the following questions. there is some rcclati onsh ip hetvvcen the two terms to the left ofi: and the same relationship holds between the two terms to its right. Also, ill each question. one term either to the right 01 :: or to the left of i t is missing. This term is given i.IS011e of the alternatives given below cnch quc st ion. find out this term.

IAG.IU 10K: '!

(a) LR (1)) MS

(e) PY

(d ) SY

2. ACE: Fill .: OQS:')

(a) PRT (b) RTU (e) TVX

(d) UWY

3Q

Find the 000 MAN OUT
al IRJO bl UFVE C) SHTG d) NMOK
01 MRSN bl RWXI c ) KQSP d) AFGZ
a) HKNP b) KNOT CI MPSV d) PSVY
a) LPWG I)) XBIS c} FJQB d) MOXH
a'i AOZI b , PJKO c) FJQB dl FRUI
a) AFWB t» XCUY C) KPGZ d) RWNS
a) ovci hi RYFM c) KRYF d) MTAH
a'i ADIP RULC; c ) ORWF d) PSXE
a: LPVD b) KOUe c ) BFI_V d) OSYG
10 " CJTG I)) NUCF<1 c ) QXHW d) PWHi
11 a) JA10L bl MCVI/RO C) VMFAX d\ OGYTQ
12 a) RVCNZ hi l P'NG r c) OSZJW d\ BFMWJ
13 a) KPTGXll hi OJVEZA c ) GTHSYB dl FUJOOl.
14 a) GJMPS bl RUX,\B CI MPSVY dl LORUX
1 S a) RWBGL bi FULEJ CI EJOTZ dl OTYDI
1G ') DIRWFk bi MRAFOT ci RWFKfY dl HMU<:IN
1l a) BEJMR\j hi f-:J:-;XAFI C) WZEHMP d) ZCHKQT
18 a) VLDXTR bl Pr-:-XRNL c) SXAUQO dl GWDIEC
19 a} LOTA,ILJ b) LJXCJTD CI DGLSBM dl ZCHOXI
20 a) GKOSXB bl r,,::vZDHL cl YC;GKOS d) MQUVCG
21 a) RWNS,]p bl OTKPGL ci EJAFWB dl AFWBSX
22 a) IQJROI b) FfGUDW ci LLJAZBT dl EUFVGS
23 aJ LQVAFK b) OTYDIN C) EJOTYE d) AFWBSX
24 al KPZATG bl YBWDTG C) XAVYTW d) VESHLM
25 al MPKNIL bl ORMPKN c) XAVYTW d) CFADXB
26 a) LlFCY b) ROLIF C) MJGOA d) GDAXU
27 a) PTAKX bl UYFPD c) ZDKUKH d) MQXHU
28 a) RLFZT b) OICWP c) MGAUD d) ZTNHB
29 al RMHCl( bl OJEZLJ c) YTOJO d) IDYiO
30 a) AWSOK bl RNJF~ c) ZVRNJ d) OKGCX CODING - DECODING

A con..: is u "system of signals' Therefore. Coding. is,\ method oftransruiuing a message between the sender nnct the rec .. -ivcr without r. third person kllC!willb"- it

The ('odinf:!, and Decoding Test is set up 10 judge- the c andid ru c S obililY to decipher the rule that codes ,I particular word/message and break the code to decipher the message.

TYPE I : LETTER CODING

In these questions. the real alphabets in a word arc replaced by certain other alphabets according to a specific rule to form its code, The candidntc is required to detect I he common rule and answer the questions accordingly.

Case I. To form tile code for a nath er word (CODlNG)

Ex.L Ifin u certain language MYSTJI:Y is coded as N7TIJJG2. how is NEl\1ESrS coded in that Ianguagc ?

a) MDLlIRDR

b) OFNfT.I'I c) ODNI·ITDR d) POOKU(;U

Sol. Clearly, each Jetter in tile word MYSTfF,' is moved one step forward to ohtnin the corresponding lcucr ofthe code.

SYSTIFY ~

ZTlIJGZ

So, in NloMESIS, N will be coded as 0, E as F, M as N and so 011. TllUS, the code becomes OFNFTJT.

Hence. the answer is (b),

Ex.2. IfTA1' is coded as SZO. then how is FREEZl, coded"

a) EQDFYG b) ESDFYG c) GQFDYF d ) EQDDYD

Sol. Clearly, each letter in the word TAP is moved one step backward to obtain the corresponding letter of the code.

Gzo

l'

A P

Thus. in FREEZE, F will be coded as E. R as Q. E as D and Z as Y. So, the code becomes EQDDYD.

Hence, the answer is (d).

Ex.3. In a certain code, STKKIM is written as THUJL. How is TRAINING written in that code?

a) S(lI3HOHOH d) UQBJOHOH

b) UQBHOHOF e) None of these

c) UQRJOHHO

Sol. Clearly, the letters in the wordSIKKIM are movedalternatively one step forward and one step backward to obtain the letter of the code.

31

So, in TRAINING, r will be coded as, U, R as Q, /\ as B, I as 11, N as 0 and so on, Thus, the code becomes UQBlIOHOr,

Ex.4_ In a certain code, MENTION is written as LENl"!'NO. How is PATTERN written in that code?

0) APTTREM d)OTAFTRN

h) l'TAETNR

e) None of these

c) OTAETNR

Sol. Clearly, to obtain the code, the first letter of the word MENT10N is moved one step backward and the remaining letters are reversed in ordr,

taking two at a time. ",....--,

L___+l

M ~ L

n

So, in PATTERN. P will be coded ('IS 0 and the sequence ofthe remaining letters in the code would he TAETMR, Thus. the code becomes

OTAETNR

Hence. the answer is (c).

Ex_5. ln a certain code. FORGE is written as FPT.ll. How is Cl ILPRIT written in that code '!

a) CSJNPUR

b) CVMQSTU e) CVNSVl\i!

d) cxosur. \V

Sol. Clearly, the tlrst letter in the word FORGE remains as it is and the second. third. fourth and fifth letters are respectively moved one. two, three and four steps forward to obtain the corresponding letters of the code

Applying the same rule to the letters of the word CULPRIT, C will remain unchanged, U will be coded as V, L as N. P as S, R as V. I as Nand T as Z. Thus, the code becomes CVNSVNZ.

Hence, the answer is (c).

Ell.6. lfin a code, ALTERED is written as ZOGVIVW, Then in the same code, RELATED would be written as

a) IVOZGVW b) IVOZGWV c) IVOGZVW d) VIOZGVW

Sol. Clearly. each letter .. fthe word AL TBRED is replaced by the letter which occupies the same position from the other end ofthe English alphabet, to obtain the code. Thus, A, the first letter of the alphabet. is replaced by Z, the last leiter. L, the 12th letter from the beginning of the alphabet, is replaced by 0, the 12th letter from the end. T, the 7th letter from the end of the alphabet is replaced by G, the 7th letter fr()m the beginning of the alphabet, and so On.

Similarly, in the word RELATED, It will be coded as J, E as V, Las C, A asl" T as G and D as W. Thus, The code becomes JVOZGVW. Hence, the answer is (8).

32

TYPF: : Nt/MilER CODING

In these questions. either numerical code values art." assigned to a word or alphabetical code letters arc assigned to the numbers. The candidate i~ required to analyse the code as per the directions.

Case] : 'Yllell n umericat code values ,""I;! «, ... dglle" 10 words

Ex.L. I f in a certain language A. IS coded as I, B is coded as 2. and so on. how is BlDDIC coded in that code?

a) 29449J h) 28456~ c) 375582 d) :F)44()2

Sot. A~ given the letters arc coded as

1\ II C [J I 3 4

E G II I

5 6 8 l)

So. in BlDDIC, B is coded as 2, I as 9, D us 4. aud C as 3. Thus, BIDlJIe is coded as 294493.

Hence. the unswc- is (a),

Ex.2. I f PAINT is coded as 741:?S and EXCFI. us coded as f)])96. then how would you encode ACC'FPT?

0) 455978 h) 547978 c) 55497R d ) 7]5%1

So1. f'Icurly, in the given code, the alphabets are coded as follows:

PII.INTFXCI. 7412R9356

So. in ACCEPT, A is coded us 4. C <\~ 5, E as 9. P us 7 and T ~\S R. Hence, the correct code is 455<)7R and therefore. the answer is (a), EX,3, II D ~ 4 and COVER ~ 63, then BASIS -::

a) 49 b) 50 0.1 54 d) 55

Sol. Clearly, in the given code. A = 1.13 2. C - .'.. . so that COVER = 3 ~ 15 + 22 + 5 + 18 = 63.

Now, in BASIS, B = 2, A _, I, S 19, I - 9. Thus, BASIS = 2 + I + 19" <) + 19 = 50. Hence, the answer is (b).

TYPE: SlJBSTlTUTION

In this type of questions, SOme particular objects arc assigned code names. Then a question is asked that is to be answered in the code language Ex.I. If cook is called butler. butler is called manager. manager is called teacher, teacher is called clerk and clerk is cailed principal, who will teach in a class"

a) Cook b) Butler c) Manager d) Teacher c) Clerk

Sol. Clearly, a 'teacher' teaches in a class and a~ given "teacher" is called 'clerk'.

So, a 'clerk' will teach in the class.

33

Ex.!. If diamond is called gold, gold is called silver. silver is called ruby and ruby is called ernrald. which is the cheapest jewel ?

a) Diamond b) Silver c) Gold d) Ruby c) Emerald

Sol. We know that <silver' is cheapest. But, as given. 'silver is called 'ruby'.

So, 'ruby' is the cheapest. Hence. the answer is (d).

Ex.3. Ifeye is called hand. hand is called mouth. mouth is called ear, car is called nose and nose is called tongue. with which of the following would a

per son hear?

a) Eye b) Mouth

0) Nose

d) Ear

e) Tongue

Sol. A person hears with his 'ear'. But as per the given information. 'ear' is called 'nose'. So, a person will hear with the 'nose'.

Hence, the answer is (c).

TYPE: MIXEO LETTER CODING

In this type of questions. three or four complete mexsagcs are given in the coded language and the code for a particular word is asked. To anul yse such codes, any two messages bearing a common word arc picked up. The COmmon code word will mean that word. Proceeding similarly by picking up all possible combinations of two, the entire message can be analysed.

Ex.1. If "n.\'O ptr kl i elm' stands for 'Shcirma gels marriage KUi' pI I' l nm wop chn stands for 'wife gives murrui ge Kdi', 'tt; wop nhi ' stands for 'he gives nothing', what would mean 'gives'?

a) chn b) nhi 0) ptr d) wop

Sol. In the second and third statements, the common word is "gives' und the common code word is 'wop', So, 'wop' means 'gives',

Hence, the answer is (d).

F.x.2. 1 f 'tee see pee' means 'Drink fruit juice' ~ 'see kee lee' means 'Juice is sweet' and 'lee ree rnee means 'He is intelligent', which word in that language mcanse 'sweet' ?

a) see b) kce c) lee

d) pec

e) tee

Sol. In the first and second statements, the common word is' Juice' and the common code word is "see". So, 'sec' means 'Juice'?

In the. second and third statements, the common word is 'is' and the common code is 'lee', So. 'lee' means 'is'.

Thus, in the second statement, the remaining word 'sweet' is coded as 'kee'.

Hence, the answer is (b).

34

TYPE: MIXED NUMBER CODING

In this type of questions, a few groups of numbers each coding a certain short message, are given. Thought a comparison of the given coded messages. taking two at a time, the candidate is required to find the number code for rnch word and then formulate the code for the message given.

Ex.j . In a certain code, '"786' means "study very hard', '958' means 'hard wnrk pays' and '045' mcan s 'study and work". Which of the following is the code for

very

a)

b) () c) 7 d) Can't be dc tcrrui ncd c) None of these

Sol. In the first and second statements, the common word is "hurd ' and the

common code digit is '8', So. 'S' means "ha rd".

In the first and third statements, the common word is 'stud}" and the common code digit is "6'. So. '6' means 'study'.

Thus, in the firsr statcment , 7 means 'very', Hence, the answer is (c).

F,~.2. I f in a certain code language, '324' means "Light is Bright'. '()2_tj' mcau-. Gir-l is beautiful' and <4758' means '1 prefer hright clothes', which digit mcun s 'Light' in that language?

a ) 3

hi 2

c ) 4

d I 7

e) .')

Sol. In the first and second statements, the common word is "is' find th c

common code digit is '2'. So. '2' means 'is'.

In the first and third statements, the common word is "hri gh t " and the common code digit is "4'. So. "4" means "bright".

rhus, in the first statement. '3' means 'Light' Hence, the answer is (a).

35

TYPE: DECIPHERING INDIVIDUAL LETTER CODES BY ANALYSIS

In this type of questions, certain sample words arc given along with their codes.

The candidate is required to decipher individual codes for different letter by comparing. taking two words at a time, and then answer the given questions accordingly.

Example: Below, in column I, are given some words. These have bun Ir.ltslaled in 10 a code language. Tire clJtie '.,.I .... I.nt. of lit .. wnrds in column I .ivelt III tJu .ame order es tie se leite, "ce"" t. Ite Of'Wt..1 "'OI'd. Study tlte two columns carefully and Ilren of Ihe four elternatives given in e"ch queslion, find the 0"" thai h". tlte code e ., .. iv.'ent. ofl"t! l.tltlTs nftlr e w ord give" in tI,e ,,,~stio,,. Tltis is ~.r illU'tNr.

Col1UnRI

~ ... ft

DELlBERA nON CONSIDERATE GHOSTLlKE WORLDLY KNOWLEDGE ROCKET

eemrqs ccchlmc cfiuno'lQfX cdgrnqrsxz adefmopqqsz cefkrnopqqszz

1. SOLACE
a) aedpqrb) acemoq c) acdmpq d) demopq
2. KNIGHT
a) fgrsxz b) gprsxz 0) fhrnpq- d) ghrxyz
3. WORDY
a) thlmq 1:» ehlmo c) efhlm
4 NOTICE
a) acdeqsb) afrnqsz 0) efhpqs d) fghpqr
S. BLOAT
a) lkpqz b) hmpqz c) cmpqs d) adeop

d) ckrops

________ .------------

Solution: We first find the exact codes of the given words.

DELlBERA TlON is a twelve-letter word. So. its code is cefkrnopqqszz. CONSIDERATE is an eleven-letter word. So, its code is adefrnopqqsz. GHOStLIKE lind KNOWLEDGE are nine-letter wonts and the codes are

cfhrnoqqrx lind cdgrnqraxz. KNOWLEDGE has two E's and so its Code will also contain two identical letters. thus, its code is cfhmoqqrx.

So. the code for GHOSTLIKE is cdgrnqrsaz.

WORlDL Y is a seven-leter word. So, its code is ccehlmo, ROCKET is a six-letter word. So, its code is aelllrqs. Rearranging the words lind their codes, We have:

DELIBERATION CONSIDERATE GHOSTLIKE WORLDLY KNOWLEDGE ROCKET

cefkmopqqszz adefmopqqsz cdgrnqrsxz ccehlmo cfhmoqqrx aemrqs

The common letter the given words is 0 and the common code letter is m. So. m stands for O.

In WORLDL Y and ROCKET, the common code letter m stands for O. The other common code letter c stands for R

In GHOSTLIKE and WORLDLY, the other common code letter c stands for L. In DELIBERATION and WORLDLY, the common code letters, c, c and m stands for L, Rand 0 respectively. The other common code letter 0 stands for D.

In WORLDL Y and KNOWL EDGE, the common code letters, c. e and m stand for 0, Land 0 respectively. So. the other common code letter h stands for W.

In KNOWLEDGE, there arc two E's and the letter q occurs twice in the code.

So. q stands for E.

In KNOWLEDGE and ROCKET, the common code letters m and q stand for 0 and E respectively. So. the other common code T stands for K.

In GHOSTl.IKE and ROCKET, the common code letters, m, rand q stand for 0, K and E respectively, So, the other common code letter s stands for T.

In ROCKET, the remaining code letter a stands for C.

In ClIIOSTLTKE and KNOWLEDGE, tin: common code letters m. c. rand q stand for O. L. K and E respectively. So. the other common code letter x stands for N.

In KNOWLEDGE, the remaining code letter fstands for N.

In DELIHERA nON and GHOSTLIKE, the common code letters '1, c, sand I1l stand for E. L. T and 0 respectively. So, the common code letter d stands for S.

In CONSIDERATE and GHOSTLIKE, the common code letter 111, z: '1 and s

stand for 0,1, E and T respectively. So, the common code letter d stands for S.

In GHOSTLIKE, the remaining code letter g stands for H.

In CONSIDERATE, the remaining code letter p stands for A. In DELIBERATION, the remaining code letter k stands for B. The information can be summarised as below:

37

1.(c):The code for Sis d. for 0 is m. for Lis c, for A is p, for C is a and for F is q So. the code for SOLACE is dmcpaq or acdmpq.]

2.(a): The code for K isr , for N is f, for I is r: for G is x. lor I-I is g and for Tis s.

So, the code for KNIGHT is rfzxgs or fgr sxz.

3.(b):The code for VII' is h. for 0 is m , for R is c, for D is 0 and for Y is L.

So, the code for WORDY is hmeo l or chlmo.

4.(b):The code for N is f. for 0 is m , for T is s , for 1 is r; for C is a and for E is q.

So, the coder for NOTICE is fmszaq or efmqsz.

5.(d):Thc code for 13 is k , for L is c , for 0 is m , for A is p and for Tis s.

So, the code for BLOAT si kemp's or ckmps.

38

If JAPAN is coded as K B Q B 0, then the code l P S F B stands for

a) K 0 REA b) K A P I L

c) K A V Y A d) K RON E

If ANIMAL is coded as M B N JOB then, how do you code JACKAL

a) K B 0 L B M b) DOL PIN

c)OINING d)DELINK

If WINDOW is coded as V H MeN V, then the code C H M H M F stands for

a) DEC 0 DEb) DOL PIN Cj DIN I N G dj DEL INK

If BOTANY is coded as X M Z S N A, then how do you code POLITY

a) X S H K N P bj X S H K P 0

cjXSHKOO c)XSHKNO

5. If TENNIS IS coded as W H Q Q L V then the code K R F N H B stands for

a) HOW R A H b) H J H 0 0 R F

c) H 0 USE S d) HER B A L

If SCHOOL is coded as 0 R R K F V then how do you code COLLEGE

a) H J H 0 0 REb) H J H 0 0 R F c) H J H 0 0 R G d) H J H 00 Q F

If RADIO IS COded as U X G F R, then how do

you code COOLER a) F L R I H P

c) F L R I H J

b) F L R I H N d) FIR I H 0

If BHOPAL IS coded as C GPO B K then the code N Z E Q B R stands for

a) MAN I I A b) M E X leo

c: MAD R A S di M 0 S COW

If TREES is cocec as SEE R T then how do you code LEAVES a) S E V A E M c)SEVAEF

b) S E V A E K d) S E V A E L

10 If PILLOW is coded as Z L 0 I L M, then the code Q F D Q U R F stands for

a) CUR T A I L b) CAN V S S

bj CUR T A I N d) C 0 CON U T

11 If SPEAK IS coded as V M H X N, then how do you code W R I T E

a) Z 0 L Q H b) Z 0 L Q I

c) Z 0 L Q G d) Z 0 L 0 J

12, If ARABIC IS coded as B Q B A J B then how do you code PERSIA

aj Q F S T J B b) 0 D S R J Z

c) Q D S R J Y d) Q FS T J A

13 If SHOULD is coded as H S L F 0 W then how do you code PRONOUN

a) Q S POP V 0 b) Q S POP V 0

c) K I L M L F M dl K I L M L F N

14. If EXPENSES IS coded as S E S N E P X E then how do YDU code INCOMES

a) S E MOe N J b) S E Moe N K

c) S M E 0 C N S d) S E Moe N I

15 If '~AGAPUR is coded K X 0 M R 0 then the code Y F E X 0 stands far

a) BID A R b) 8 I H A R

c) BOX E S d) BAS I N

16. If DHAKA IS coded as G K 0 M 0 then the code L V 00 Q G stands for

a) INC 0 M E b) INS 1ST

c) IN F USEd) I S LAN 0

17 If BANG LA IS coded as AZMFKZ. then how do

you code BARODA ar CBSPEB

01 AZQNCZ

b) BESPSBC d) AZQNCB

18 If ROUBLE IS coded as FKCTPQ then how do your code DOLLAR

a) SZMKPC c) SZMKPD

c) SZMKPB d) SZMKPB

19 If SWEDEN IS coded as VTHAHK, than how do you code NORWAY

a] QLUTOV b) QLUTDW

cl SZMKPB dl SZMKOC

20 If SHARES IS coded as RGZQDR_ than how do you dade STOCKS

a) RSNBJT bl RSNBJG

c) ONIONS d) ORANGE

21 If APPLES IS coded as ETTPIW than the code SVERIKI

a) BANANA bl GRAPES

c) ONIONS d) ORANGE

22 If LEASE is coded as HAWOA than the code LHWJP stands for

a) PLATE b) MAGIC c) PLANT dl PLANS

23 If TRAIN IS coded as XNEER Ihan the code OWKEG stands to-

a) MATHS b) MAGIC

c) MARKS d) MAKES

24 If CENTRE is coded as YAJPNA than the code OPWPAO stands for

aj STATES bl STOOLS

c) STEELS d) STEALS

25. If STEEL is coded as HGVVO than now do you code CHILD

a) XSROW b) XSROV c) XSROX d) XSORX

39

26 If FRANCE IS Coded as KWFSHJ. then the code i'KWNHG stands for

aj ALASKA b) ARABIA

c) AFRICA d) ASIANs

27 If ISRAEL IS coded as DNMV2:G then the code VHZMDXV stands for

.1 ARMENIA b) ALBAMA

CI AMERICA dl ALBANIA

28 If JOR.DAN IS coded as OJWYFI then the code GJXINV stands; for

a I BOSTON br BOSNIA

CI BASKET dl BASICS

29 If DOCTOR IS coded as EPDUPS, then how do yOl; code READER

.1 SFBEFS bl SFBI'FT

c) SFB'=:FU d) SF[BFS

30 If CHEAT IS coded as BGDZS, then how do you cooe TEACH

al SDZBG bi SDZer

SDZBH d) sszu.

31 If SHEE T IS coded as XCJZY then how do yOll code SLEE_P

:3) ZGJZU lJ) XGJZV

c) XGJZT d; XGJZW

32 If WATER s coded 3S XZUOS, then how do you code WHEAT

a) XGFZS b) XGFZU

c] XGFZV dl XGFZI

33 If WATER IS coded asOF:PZO, then how do vou code MAAlA

.1 PXDWE b) PXDWF

CI PXDWD d) PXDWC

34 If MATHS IS coded as PDWKV then how do you code ECONOMY

a) HGRQRPA b) HFRQRPC

c) hFRQRPD dl HFRQPB

35 If GLASS IS coded as DIXPP, then the code JFOOLO stands for

a) MIRAGE b) METRES

c) MISLED d) MIRROR

36 If POLICE is coded as BZFILM. then the code IXOBKBD stands for

a) GENERAL b) GAR RAGE

c) GARBAGE d) GARDEN

37 If BOOKS IS coded as ELRHV. than the codeQLWHP stands for

a) NOTES b) NOOSE

c) NOTEX d) NONE

40

38. If CABAGE IS coded asFHBCBD. then the code PUBNPU stands for

a) POTATO b) TAMATOc) THIRY d) NONE

39 If OFFICE is coded as ECIFFO, then how do you code FACTORY

al YROTCAE bi YROTCAD

c) YROTCAF di YROTCAB

40 If BUDGET is -odec as ATCFDS. then the code DWODMRD stands for

a) ENLARGE bl ENRAGE

c) ENMASS dl EXPENSE

4f If INDIAN is coded as JOEJBO. then how do yOU code KOREAN

oj LPSFBP bl LPS~BO

Cj LPsrBO d) LPSFBR

42 If AlG::BRA is coded asAGElRAB then how do you code INDICES

a) IDINESC bi IDINESA

CI IDINESB di IDINESD

43 If PENCIL IS coded as. PNCLlE, then how do you code RUBBER

a) RBERER b) RBBREU

c) RBBREUf{ dJ RBBER~

44 If TOWEL IS coded as SNVODK, trier- how de you code NAJIB

a) OBKJA h: OBKJ8 cl OBkJC dl OBKJD

45 If TOWEL IS coded as SNVOK, then how do Y0L; code BUREAU

a) ATQDZV bl ATODZW

ct ATODZIJ dl ATOOn

46 If It\lO\GO i~ coded ae JMEHHN, then how do yow code V!OLET

a) WHPKFR bl WHPKFT

c) WHPKFU d) WHPKFS

47 If IRON IS coded as NOR!. then how CIa you code STEEL

al LEETS bj LEEST c) LEnU d) LEETV

48 If WIMDOW is coded as XJOEPX. then how do you code COOLER

a) DPPMFQ b) DPPMFR

c) OPPMFS d) OPPMFQ

49. If EQUAL is coded as DPTZK. then how do you code BOXES

a) ANWDR b) ANWDS

c) ANWDQ d) ANWDT

50. If PHONE is coded as KSLMV. then how do you code TELEX

a) GVOVC b) GVOVB

c) GVOVA d) GVOVD

WhiCh of the following IS a place of worship

a) CUCHRH b) LOOTS

c) DOG d) CABS

Which of the following is the name of a subject

a) GNTSEECD bl GNTSEEIC

c) GNTSEEDE n) GNTSEEID

Which 0: the following 15 the name of an American President

a) SUHB b) NNXID

c) RETRAB dl CITfilQN

Wrlch of th€ following IS t-e name of a Prime Minister

'I JORAM 01 RONIM

c) IONIA d) HRUFN

Which of the foltowmq IS the name of capital

a) COWSOS b) IXDCEM

c ) OOSCWM jl AMANAP

Whic!"'. of the foil iWlllg the name of a

aj

ci KEUMHS

b) ORESIJ d) RAMUK

Which of the tottowmq IS the name of a superstar

al N~YRU bl HSIRA

c) BATAMI

d: RAIFAE

Which of the following is the name of a State In India

a) DHARBN hi KKIJIM

c ) LAGNEA d) TARUGAJ

'NhiCh of the following 15 Ihe name of a Nobel

Prize Winner a} HARAVVJA CI DELANAM

b) IDHAGN

dl NAMLEAD

tu

Which of the fottowinq is the name of a

currency

a) CUODSE c) NAXU

bl OLLARE d) NEX

11

Which of the following is an African Nation

a) AIREZ b) LUKASA

c) AIMBYY d) MBAMBA

12

Which of the following is the name of a Tamil King

a) AOMMAN bl MMANOC

c) OIAPANN d) VENNAI

13 Who among the totlowmq IS a well known furnster turned politician

a) RGM bl NAR

c) TUDT dj JAR

14 Which of the following is a vegetable

a) JACBRIN b] GERNIG

c) EGBAAC dl POTABOP

15 Which of the following is the name of a fruit

a) AEPLE b] ANROEG

c) MAonO dl ATPOTO

16 Which of tile following IS the name of cccldnnk

ar TUHSMPU b) STOPOLOG

C) FROOZY dl ZAAAM

17 Which of the touowmq IS the name of city

a) ORA BAD b) BCBAZY

c) MECOIY dj MOOWO

18 Which of the following is tile name of a flower

a) ROZE OJ 1.()SU f

01 MINESAK d) NONE

19 Which of the follOWing is the name of river

a) YAANUS b) LUJlUJ

c', GESNAG dl OUSNI

2C Which of the following is the name 01 an Atomic Power Station

a) TARUR o) PAKKIL

c) LISBON dl RONAR

21 Which cf the following is a planet

a] ARTHF bl TURNAX

cl NUSXRA dl lERJUIP

22 Which of the following IS a branch of commale

a) CTINGOS b) TAEXS

c) METGEOYR dl CULUSLAC

23

Which of the following IS the name of an animal

a) NTPHALEE b) KOCBULL

c) SEOMU d) NASKE

24

Which of the following IS the name of an airhner

a) ZABS b) DARUGA

c) QUANTA d) FLUTE

25

WhiCh of the following is the name of a Book

a) JJAAL bl NICSATA

c) GHDUAGH dl NONE

41

If In a certain language A 15 coded as 1, B is coded as 2. and so on, how is STAR coded in that code?

1) 1610291 3) 1920118

21 9120118 4) 1920611

If In a certain language P is coded 85 7, T as 3,0 as 4. R as 9 and A as 2 how IS PARROT coded in that code'?

1) 372994 3) 279943

2) 723349 41 729943

If In certain code A IS coded as 1, B is coded as 2. and so on, how IS COUNTRY coded In that coda?

1) 31521142182 3) 31511420~825

2) 351211420182 4) 2515211420183

If in a certain language T IS coded as 9, Gas 7, R as' I as 0 and E as' 3, has IS TIGER coded in that code?

1) 90731 3) 91073

2) 19037 4) 71903

If in a certain ianquaqe Z is coded as 0, Y as 5,0 as :2 _ G as 7 and L as 6. how is ZOOLOGY coded In that code?

1) 0006071 3) 7502262

21 0226275 4) 2026275

If In a certain code BOX is coded as 213, BITTER as 207749. how is BOXER coded in that code?

1) 21359 3) 23149

2) 22159 4) 21349

If In a csrtamcode TREE IS coded as 7100, FROG as 2159 how is FREE coded in that code?

1) 2100 3) 1003

2) 3100 4) 1002

42

If in a certain code LETTER IS coded as 527729, BETTER as 027729 and BUT as 017 how is BUTTER coded in that code?

1) 107729 3) 107792

2) 017729 4) 017792

If in a certain code GUN IS coded as 309, DEAR as 1468, how IS UNDER coded in that code?

1) 90148 3) 09148

2) 39148 4) 49138

10 If in a certain code SCHOOL IS coded as 123445 TEAM as 6078, how is HOTEL ceded in that code?

1) 34605 3) 60834

2) 43605 4) 34785

11 In a certain code 1 is coded as S. 7 as E 9 as H, 6 as L, 0 as T and 2 as 0 How IS 9612076 ceded In that code?

1) SEHLTO 3) HOSLET

2) HOSTEL

3) TOSHEL

12 In a certain code 915 coded as C, 0 as B, as E. 7 as A and 3 as L How is 97035 coded in that code?

1) LEBAG 3) BAGLE

2) GEBLA 4) GABLE

13 In a certain code Cis coded as 0, E as 7 T as 4, I as 9, Pas 1, R as 3 and U as 5 How IS 1904537 coded in that code?

1) PICTRUE 3) RICTPUE

2) PICTURE 4) PCTUREI

14 If certain language 1 15 coded as A, 2 as B, as C, and so on, how is FLOWER coded in that code?

1) 6121523518 3) 6211523518

2) 6121823515 4) 6218123515

If in a certain language 1 is coded as A. 2 as B.3 as C and so on. how is LEMON CQCJed in that code?

1) 12513014 3) 145131512

2) 1·25131514 4) 14013125

In a certain code 23 is coded as ex. 529 as FOR and Otltl9 as DOOR, then how is :.::35290 coded?

1) DOXFOR 3) OXFORD

2) FORDOX 4) FOXORD

In a certain code 2468 rs coded as PART. 8136 as TOUR, then how In 246618 coded?

1) PORRAT 3) PARROT

2) TARROP 4) RAPORT

In a certain code 01234 IS coded COUNT. 6785 TRAY then how IS 0123475 coded?

1) COUNTRY 3) RYCOUNf

2) TRYCOUN 4) YRCOUNT

In a certain code 13352 rs coded as OFFER. 795 as ICE. then how is 0123475 coded?

1) FICEROF 3) OICFFER

2) CIFFOER 4) OFFICER

20 In a certain code 1223 15 coded as Book. 627962 as TOMATO. then now IS 126627 coded

1) BOTTOM 3) TOBOMT

2) MOT fOB 4) BOMOTT

21 If BOOK IS coded as 43, what will be the code number for PEN?

1) 53

2) 33

3) 35

4) 43

22 If TOWER is coded as 81 what wili be the code number of POWER?

1) 75

2; 55

3) 18

4) 77

23 If OX IS coded as 39 what wdi be the code number for LION?

'1 20

2) 25

3) 38

41 50

24 If FOX IS coded as 45 what will be the code number for BOX?

1) 41

2) 49

3) 55

4) 50

25 If MAN is coded as 28 what will be thecode number for CHILD?

1) 25

2) 36

3) 49

4) 64

43

If 'light' IS called 'dark', 'dark' IS caltec 'green', 'green' 1$ called 'blue' 'blue' IS called 'red', 'red' IS called' white' and 'white' is called 'yellow', what IS the colour of blood?

1) Red 2) Dark

4) Yellow

3) White 5) None

If 'water' IS called 'black' 'blaCk' IS called 'tree', 'tree' IS called 'blue' 'blue' IS called 'ram' 'rain' is called 'pink' and 'pink' IS called 'fish' in a certain language What will the colour of the 'sky' be called In that language?

1) Blue 4) Pink

3) Rain 5) None

2) Fish

If 'black' means 'wrute' white' means 'red' 'red' means 'yellow' 'yellow' means 'blue' 'blue' means 'green' 'green' means Violet and violet means organge. what IS the colour of the sky?

1) Green 2) Violet 4) Yellow

3) Orange 5) Blue

lf'rains' IS called 'pink' 'pink' as called 'cloud' 'cloud' is called 'water' 'water' IS called 'breeze' and breeze IS called 'moon' wahl do you wash your hands .n ?

1) Water 2) Rain 4) Moon

3) Breeze 5) None

If the animals who can walk are called 'swimmers' animals. who 'crawl' are called 'flying', those who lives In water are called 'snakes' and those who fly in the sky. are called 'hunters' then what will a 'lizard' be called?

1) SWimmer

3) Hunter 4) FlYing

2) Snake 5) None

If 'cloud' IS called 'white' 'white' IS called 'rain" 'rain' Is-called 'green' 'green' IS called 'air', 'air' IS called 'blue' and 'blue' ts called 'water', where do Ihe birds fly In?

1) Air 4) Rain

2) Cloud

3) White 5) Blue

If 'air' IS called 'water', 'water' is called 'green', 'green' IS called 'dust', 'dust' IS called 'yellOW' and 'yellow' IS called 'cloud', which of the following do fish tive in?

1) AI[ 2) Water 3) Green

4) Dust 5) Yellow

If 'Wind' IS called 'stone', 'stone' IS called 'water', 'water' is called 'breeze', 'breeze' is called 'rain', 'rain' is catled 'hal' and 'hot' IS called 'cold' In a certain language, What do we wash our hands With' In that language?

1) Water 2) Breeze 3) Cold 4) Stone 5) None

If 'dust' is called 'air', 'air' IS called 'white' 'white' is called 'yellow', 'yellow' is called 'water' and 'water' IS called 'red', then where do the birds fly In?

1) Yellow 2) White 3)Alr4) Water 5)Green

10 If 'water' IS called 'food', 'food' is called 'tree' IS called 'well', 'well' is called 'sky', on which of the follOWing grows a flower?

1) Water 2) Food 3) Tree 4) Sky 5) Well

11 If 'rain' 15 called 'water' 'water' is called 'road' is called 'cloue, 'ctoud' IS called 'sky' 'skY'ls called 'sea' 'sea' IS called 'path', where does the aeroplane fly?

1) Cloud 2) Water 3) Road 4)Sea 5)None

12 If 'sky' is called 'sea' 'sea' is called 'water' 'water' IS called 'air', 'air' IS called 'cloud' and 'cloud' is called 'river' then what do we drink when thirsty?

1) Sky 2) Air

3) Water4) Sea 5)Cloud

13

If 'pen' IS called 'pencil' is called 'scale' 'scale IS called 'bag' and 'bag' is called 'book', which is used 10 carry the books?

1) Scale 2) Pen 4) Bag

3) Book 5) Pencil

14 If 'room' IS called 'house', 'house' IS called 'roof 'roof' is called 'floor', 'floor' is called 'terrace' What will a person stand on?

1) Floor 4) Room

2) Terrace 3) Roof 5) House

15

If 'paper' is called 'eraser', 'eraser' is called 'bag', 'bag' is called 'scale' 'scale' is called 'pencir and 'pencil' is called 'paper', what Will a person write with?

1) Pencil 2) Paper 4) Bag

3) Eraser 5) Scale

In a certain code language, "Col t.p mol' means 'singing IS apprecibble', 'mot baj min' measn 'dancing is good' and 'lip nap baj' means 'singing and dancing' Which of Ihe tollowmq means 'good' In that code language?

1) mot 3) baj 5) None

2) min

4) Csnnotbedetemmed

In a certain cod 'down sky' IS written as 'zoe pun' and 'on top sky' IS written as'pun ta rep' How Will 'down on' be written in that code?

1) zoe ta

3) rap zac

4) None

2) pun rap

4) Dala Inadequate

In a certem code language, ' dam pul ta' means 'bring hot food 'pultir sap' meaos 'food IS good' and 'tak ca sap' means 'ggad bright boy' Which of the tollowm does means 'hal' In that language?

1) dam 31 ta

5) None

2) pul

4) cannal be determined

In a certain code language 'ken pot I' means 'ggod morning' 'hu snanq' means 'come on' and 'nu ken sue' means 'come for good' Which worcsn that language means 'for'?

1) shang 3) sue

5) pot:

21 ken 4) hu

In a certain code language 'pul ur fin' means 'ggad sweet truit' 'tie dip sig' means 'beautiful red rose', '519 Ion fin' means 'rose and tnnt' Which of the fallOWing stands for 'and' In thai language?

1) pul 3) Ian

5) None

2) tir 4) slg

In a certain code language 'pul ta nop' means , fruit' IS gOOd' 'nop ka tir' means 'tree is tall' and 'pul ho sop' means 'eat goad food' Which of Ihe followinq means 'fruit' in Ihat language?

1) pul 3) nop

5) None of these

2) la

4) Date inadequate

In a certain code 'XlM' means 'He IS bright' TelO means 'Every lawn IS green' and 'OQCN means 'Every wall was green Which of Ihe Iollowinq means 'Every lawn IS bright' 10 Ihal code?

2) CXlT

41 Cannol be detennlned

1: ZTOM 3) XOTZ 5) None If In a certain language 'mxy das ZCI' means 'go0d little frock' 'jrnx cos lCI' means 'girl behaves goad' nvg drs cas' means 'gill makes mischref and 'cas aJP cas means 'little gtrl fell' Which word In Ihallanguage stands for 'frock'?

1) z ci 3) nvg

5) None of these

2) das 4) ajp

In a certain code 'trr me sac' means 'green and tasty', 'drc sac gar' means 'tomato IS green' 'voc III for' means 'food IS tasty' Which of Ihe Iollowinq means 'tomato .s tasty':n that code?

1) lor til cic 3) Til me for 5) None

2i die for sac

41 Cannol be celerrrmed

10 In a certain language 'pre not bis' means 'smoking IS harmful' 'vag dar nor means 'avoid harmful habit' and 'dar brs yel' means please avoid smoking' Which of Ihe fallOWing means 'habit' In that language?

1) vag 2) not

4) Cannol be determined

3) dor

5) None of Ihese

11 In a certain code language '3a 2b, ie' means 'Truth IS External' "ti: sa. 8b, 3a means 'Enmity IS not eternal' '9a,4d 2b, 8b' means 'Truth does nat perish' Which of the fallawrng means 'eternal' In Ihat language?

1) 3a 2) 2b

4) Cannot be determined

3) t«

5) None of these

12 In a certain code language 'Ka BI Pu' means 'You are mtelliqen:' 'Ya la Ka Wo' means They seem very intelliqent' la Pu le' means 'You can see' and 'Sun Pun Yun Ya' means 'hOW Intelligent she IS' In thai language which of the fallowing means 'are'?

1) Ka

2) 81

3)le 4)Pu 5)Ya

13 In a Cerlaln cede language "Tlnk Log Se' means 'Flu;ls are Tlpe' 'Se Thao Hay Tink' means 'Mangoes are not nps' 'Hay Se Cue Tlnk' rneans 'Bananas are not ripe' Which word In that language means 'Mangoes'?

11 Tlnk 21 LCo9

31 Se 41 Tnao 51 Cue

14 In a Certain cede 'prrn to sunk' means 'red beautiful flower' 'suk torn no' means 'white and red' and 'dam 10 huk' means 'cloth IS beaullful' Which word In that \<'1nguage means 'flower'?

II pll\1 21 to

31suk 4lto", Sj None

15 In a certain \3nguage 'Cui max dir means 'nice little boxes' 'S'.lt med blx' means "well arranged row' 'bix fac oir means 'row of boxes' Which of Ihe fOllowl11g stancs 'or 'of' m that language?

Sl Iac 4)sut 5) None

16 In a Cerlall118ngua~e 'cui max dir means 'nice little boxes' 'Slit rnec bix' means 'well arranged row' 'on tJC dlf means 'row cf boxes' Which of the followtno represents 'apple' In that

language? ~

1) nae 21 tern

ptt 4) ho 5) ka

17

In a certain code language 'lee SU Jak' means 'he eats apples' jak uc' measn 'sweet orange and 'lee ce e jO tic' means 'menan eats sweet bananas' In thai language WhiCh 01 the following means 'oranges'?

1) IIC 11 lee

4) Cannot be determined

3) kee

5) None 01 Ihese

18

In a ceriau: language 'Sau Pey Te' means 'Doctor Blned Kumar' 'Ting Pu Sau' means 'Sal>sh IS Doctor' 'Ping Pong Tlng' means 'Salish and Ram' Which word In Ihal language means 'IS'?

I)Sau 21Pey 31Pu 4)1,"g 51None

19 In a certain code language 'Alaha-Beta-Phaf means 'Machine is running', 'Phi-KappaThelaAlpha' means 'Bus IS not running' and 'TheatPhi-Gamma-Alpha' means 'Car IS not running' WhiCh word 10 that language means 'Bus'?

1) Alpha 21 Beta 3) Pha,

4) Kappa 51 Gamma

20 In'" certain cor'e language 'quda uka' means 'clear water', 'pin gala' means 'overcast sky' and 'pin sal buka means 'Clear blue sky' Which word In that language means 'blue'?

Ilguda 21buka 31P,n 4)gola Sisal

21 In a certain code language, 'lin nac means 'blue shirl' 'it mit tim' means 'shirt and pant' and 'nac pit' means 'blue pant' Which word In that language means 'and'?

1) tim 21 ptt

3i nac 4) mil 5) None

22. In a certain code language 'Pat Zoo Sun means 'Eat Good Mangoes' 'Pus Slin Trrn' means 'Mangoes and Sweets' and 'Tim Zoo KIt' means 'Purchase Goad Sweets Which word In Ihe langu.ge means 'Good'?

1) Zoo 21 pus 3)51", 41Tlm 5)KII

23 In a certain cede lahguage 'Kew xas huma de~,o' means 'she IS eating apples' 'Kew tepa qua' means' she sells toy a and 'sui time cekc' means 'I like apples' Wh!Ch words In Ihe language means 'she' ans 'spcte s'?

1) 'xas' and 'deko 2) 'deko' and 'I~po'

3'1 'kew and 'deko' 41 'xas' and 'kew'

51 lew and 'xas

25 In a certain code language '11m suk ta' means 'boys are clever' and 'pu ne ta' means 'boys and girls' Which word In Ihat language means 'boys'?

1) lim 2) suk 3) la 4) pu 5) ne

In a certain code '289 means 'read from paper', 276 means 'tea from field' and '85' means 'wall paper' Which of the following IS the code for 'tea'?

1) 2 2) 6 4) Either 2 or 6

3) Either 7 or 6 5) Either 2 or 7

In a certain code lanquaqe '123' means 'hot filter coffer' '356' means 'very hot day' and '589' means 'day and 1lght' Which digit In that language means \'ery'?

1) 8

21 6

3)9

415

5) None

In a certain code '721' means 'Good College life', '526' means 'you are good' and '257' means life are good' which digit stands for 'youin the code?

1) 6 2) 5

4) Canna: he determined

3)7

5] None of these

In a certain code. '3S7' means 'get me toy', '843' means 'br.nq good toy' and '746' means 'bllng Me water' Which of the following digits represents 'good' In that code?

1) 7 1! 6 31 Date Inadequate

4'1 4 51 None

In a certain code '786' means 'bring me apple', '958' means 'peel green apple' and '645' means 'brrng green fruit' wruchot the follOWing IS the code for 'lle'?

1) 8 2) G

4) Cannot he determined

3) 7

5] None

In a certain code 786' means 'study very hard' '958' means 'hard work pays' and '645' means 'study and work' wlucho f the Iollowinq is athe code for 'very'?

1) 8 2! 6

4) Cannot De determined

3) 7

51 None

In a certain code '37 means 'which class' and '583' measn 'caste and class' What IS the code for 'caste'?

1) 3 2) 7 4) Either 5 or 3

3) 8

5) Either 5 or 8

In a certain code '247' means 'spread red

carpet' '256' means 'dust one carpet' and '234' means 'roll red carpet Which digit in that code means 'dust'?

1) 2 4) 6

2) 3

5) Cannot say

3)5

In a certain code language '526' means 'sky IS blue', '24' means 'blue colour' and '436' means 'colour IS fun' Which digit In that language means 'fun'?

1) 2

2) 3

4) 5

5) None

3)4

10 In a certain code '256' means 'red color chalk', '586' means 'green colour flower' and '245' means 'white colour chalK' What digit In that code means 'white'?

1) 2

2) 4

3)

5) None

4) Cannot be determined

11 In a certain code '467' means 'leaves are green', '485' means 'green IS good' and '639 means 'they are playinq' Which digit stands for 'leaves In that code?

1) 4

216

5) None

3)7

4) 3

12

In a certain code '256' means 'Boys are good' '637' means 'Arnar IS good' and '348' means 'Lata IS bad' Which digit means 'Amar' In that code?

1) 2

2) 7

3)6

4) 8

5) None

13 In a certain code '256' means 'you are good' '637 means 'we are bad and '358' means 'ggod and bad' Which of the follOWing does represent 'and' In that code?

1) 2

2) 5

3) 8

4) 3

5) Nore

14

ln a certain code '253' means 'books are old', '546' means 'man IS old' '378' means 'buy good books' What does 'are' mean In that code?

1) 2

2) 5

3)4

416

5) 9

15 In a certain code '678' means 'how are you', '347' means 'how IS life' and '569' means 'you were wonderful' Which numeral In tha; language means 'you'?

1) 2

2) 6

4) 9

5) None

3) 7

16 In a certain code language '743' means 'Mangoes are good' 657' means 'Eat good food' and '934' means 'Mangoes are ripe' Which digit means 'ripe' In that language

1) 5 4) 7

21 4 319

5) Cannot be determined

17 In a certain code 18nguage'134' means 'Good and Tasty', '478' means 'see Good Pictures' and '729' means 'Pictures are faint' Which of the follOWing numeral symbol stands for 'see'?

1) 9

2) 2

4) 5) None

3) 1

18 In a certain code tanguage '123' means 'brlghl little boy' '145' means 'tall big boy' and '637' means 'beautifuluftle flower' Which numeral 10 that language stands lor 'bright'

1) 1

0) 3

4) 4

5) None

21 6

19 In a certain code '123' means 'speed and accideot'. '345' means 'speed IS cause' anc '146' means 'accident IS effect' Which of the following numbral symbols stands for 'cause'?

1) 3

2) 4

3) 5

5) 2

4) 6

20 In a certain code '678' means how are they' '347' rreans 'how IS life' and '569' means 'Ihey were wonderul' Which numeral In that language means 'they'?

1) 6

2) 7

3) 8

5)4

4)3

BLOOD RELATIONS

III these tests, the success of a candidate depends upon the knowledge 0(' the hlucd relations. some of which arc summarized be-low to help solve these tests.

Mother's or Father's son Mother's or Father's daughter Mother's or Father's brother Mother's or l;ather's Sister Mother's or Father's father Mothcrs or Father's mother Sons wife

Daughrcrs husband

l lusbands Of Wife's sister Husband's or Wifcs brother Brothcrs SI..111

Brother's daughter

L nc Ie or aunt' s son Of daughter Sister's husband

Brother's wire

Grandsons or Grand daughter's daughter

Brother

Sister

Uncle

Aunt Grandfather Grandmother Daughter-in-law Son-in-law Sister-in-law Brother-in-law Nephew

Niece

Cousin Brother-in-law Sister-in-law

Great grand daughter

TYPE 1 : IH;CIPHERING JUMBLED UP DESCRIPTIONS

In this type of questions, a round-about description is given in the form of certain small relationships and direct relationship between the persons concerned is to be deciphered.

Ex.I.Pointing to a photograph. a man said, '" have no brother or sister but that mau '.s father is my father's son". Whose photograph was it ?

a) His own b) His son's c) His father's

d) His nephew's d) None of these

Sol, Since the narrator has no brother, his father's son is he himself. So, the man who is talking is the father of the man in the photograph or the man in the photograph is his son. Hence, the answer is (b),

Ex,2.Anil introduces Rohit as the son of the only brother of his father's wife. How is Rohit related to Ani! ?

a) Cousin b) Son

c) Uncle

d) Son-i-Iaw e) Brother

Sol. The relations may be analysed as follows:

Father's wife - Mother; Mother's brother - Uncle; Uncle's son - Cousin. So, Rohit is Anil's cousin. Hence, the answer is (a).

49

Ex.3.Pointing towards a person in a photograph, Anjali said, "He is the only son of the father of my sister's brother". How is that person related to Anjali ?

a) Mother b) Father c) Maternal Uncle d) Cousin e) None of these

Sol. The relations may be analysed as follows:

Sister's brother - Brother; Brother's father - Father. Father's son- Brother. So.fhc person in the photograph is Anjalis brother.

Hence, the unswere is (e).

Ex.4.Pointing out to a photograph. a man tells his friend, "She is the daughter of the only son army father's wifc't.How is the girl in the photograph related to the man?

a) Daughter b) Cousin c) Mother d) Sister e) Niece

Sol. The relations may be analysed as follows:

Father's wife - Mother; Mother's only son - Himself.

So, the girl is man's daughter. Hence. the answere is (a).

Ex.S.X introduces Y saying, "He is the husband of the grand daughter of the father of my father". How is Y related to X '!

a) Brother b) Son c) Brother-in-law d) Nephew e) Scrn-i n-l aw

Sol. Till: relations may be analysed as follows:

Fnthcr' s rather - Grandfather: Grandfather's Grand daughter - Sister: Sister's husband - Hrother-j n-Iuw. So, Y is X's brother-in-law.

! knee. the answc is (c ).

EX.(,. Poiming out to a lady, Rajan said. "She is the daughter of the woman who is the mother of the husband of my mother" Who is the lady to Rajan?

a) Au nt b) Grand Daughter c) Daughter d) Sister Sol. The relations may be analysed as follows:

Mother's husband - Father; Father's mother - Grandmother; Grandmothcr : s

c) Sister-in-law

daughter - Father's sister; Father's sister - Aunt.

So, the lady is Rajans aunt.

lienee, the answer is (a).

TYPE2: RELA nON PUZZLE

In this type, mutual blood relations or other informations of more than two persons ure mentioned and information about any two is mentioned.

Ex.I.A and B arc brothers. C and D are sisters. A's son is D's brother. How is B related C?

0) rather b) Brother c) Grandfather d) Uncie e) None of these

Sol. Clearly. B is the brother of A; A's SDn is D's brother. This means D is the daughter of A. Since C and D arc sisters. C is also the daughter of A.

So. B is the uncle of C. Hence, the answer is (d).

Ex.2.Given that

1. A is the mother of B: 3. D is the brother of E;

2. C is the son of A:

4. E is the daughter ofB.

50

The grandmother of D is

a) A b) B c) C d) D c) E

Sol. D is the brother ofE and E is the daughtcr of'B. This means that D is the son ofB, Also, A is the mother of'B.

So, A is the grandmother ofD. Hence, the answer is (a).

ExA.A and A arc married couple. X and Yare brothers. X is the brother of A. l low is Y related to A ?

a) Brother-in-law

b) Brother

c) Son-in-law

d) Cousin c) None of these

Sol. A and B are husband and wife. Since X and Yare brothers, und X is the brother of A, Y is also the brother of 11.. Tints. Y is the brother-in-law of A.

Hence. the answer ix (a).

Ex.5.Rcud the fol1()\ ..... ing Information care fully and answer the questions given below There arc six children playing football namely j\. H, C. D. E and F /\ and E arc brothers. l- is the sister of E, (' is the only son of A's uncle. Band D arc the daughter of the brother orc's father.

How is (' related to F 'I

a) Cousin b) Brother e) Son d) Uncle

2. How many male players are there?

a) One b) Three c) Five d) Six e) Four

3. l low many female players are there?

a) Two b) Three 0) Fived) Onee) Four

4. How is D related to A ?

c) None

a) Uncle b) Sister

c) Niece

d) Cousin

e) None

Solutions:

I. F is E's and hence A's sister. So, C is also the son ofF's uncle and is. therefore, F's cousin. So. the answer is (a).

2. As given, A and E are brothers. Hence, both are males. F is the sister ofE and hence female. C is the son and hence male. Band D are daughters and hence female. Thus, there three males. So, the answer is (b).

3. Clearly. from the solution of2, we find that there are three females. So, the answer is (b).

4. Clearly, D's father is the brotherofC's father and C's father is A's uncle. So, D's father is also A's uncle. Thus, D is A's cousin. Hence, the answe is (d).

51

TYPE3: CODED RELATIONS

In such questions. the relationships are represented by certain codes or symbols such as +. -, X. ~,"',O ctc., Then relationships between certain persons. given in the form of these codes, are tobe analysed.

Ea.Llf A + B means A is the sister of'B: A-B means A is the brother ofB: AxB means A is the doughter of'B, which ofthe following shows the relation that E is the maternal uncle ofD?

a) D + F x E b) D - F x E

e) [) x F + E d) D x F-E e) None of these.

Sol. Clearly. E is the maternal uncle otD means D is the doughter of the sister (say f)

ofELe. Dx F +E.

Hence. the answer is (c)

Ex.2.Read the following information carefully and answer the questions that follow:

;\ + B means A is the son of'B; A ~B means A is the wife of B: A x B means A is the brother ofB: A B means A is the mother ofB and /\ B means A is the sister orB.

I. What does P + R - Q mean 0
a) O is the father ofP b) Q is the Son 01' P.
0) Q is the uncle of P, d) Q is the brotner otP.
0 What does P x R.H-Q mean?
a) P is the brothr of R. b) P is the father of Q
c) P is the uncle ofQ. d) P is the nephew oro.
'- What does P ,"-. R t- Q mean?
a) I' is the aunt of (J. h) P is the daughter oro.
c) P is the niece arQ. d) P is the sister of Q.
4. Whuj docs P = R ~Q mean?
a) I' is the aunt ofQ. b) [> is the sister of Q .
c ) Q is the niece ofP. d) Q is the daugher of 1'. Solution;

Clearly. P '+ R - Q means P is the son of R who is the witee of 0 i.e. Q is the father ofP.

Hence. the answer is (a)

2. p x R Q means P i& the brother of R who i~ the mother of Q i.e. P is the uncle of O·

SO. the answer is (~)

52

'She is the grand daughter of the elder brother of my father' How is the gIrl in the photograph related to this man?

a) Niece b) Sister

c) Aunt

e) Maternal aunt

d) Sister-in-law

A woman introduces a man as the son of the brother of her mother How IS the man related to the woman?

a) Nephew c) Cousin

e) Grandson

b) San d) Uncle

WhIle walking with his friend Mahesh meets another man whose mother is the wife of Mahesh's father's only son, How is the man related to Mahesh?

a) Son

c) Cousin e) Father

b) Nephew d) Uncle

4, A man said to a lady 'Your mother's husbands sister's my aunt'

a) Daughter b) Grand-daughter

c) Mother d) Sister

ej Aunt

A girl introduced a boy as the son afthe daughter of the father of her uncle, The boy is girl's

a) Brother c: Uncle e} Nephew

b) Son

d) Son-in· law

If X is the brother of the son of Y's son, how IS X related to Y ?

a) Son b) Brother

c) Cousin e) Uncle

d) Grandson

lntroducinq a man to her husband, a woman saId his brother's father- is the only son of my grandfather How IS the woman related to this man?

a) Mother b) Aunt

c) Sister

e) Grand mother

d) Daughter

Pointing out to a lady a girl said, 'She is the

daughter-in-law of the grand mother of my father's only son, How is the lady related to the girl?

a) Sister-In-law c) Aunt

e) Cousin

b) Mother

d) Mother-In-law

Rita told Mani 'The girl 1 met yesterday at the beach was the youngest daughter of the brot-er-m-tew of my friend's motner How IS the girl related to Rita's friend ')

a) Cousin c) Niece e) Aunt

b) Daughter d) Friend

10 If B says that his mother IS the only daughter of A's mother, how is A r etatec to B ?

a) Son b) Father

C) Brother d) Grandfather

e) Uncle

11

Ramesh told Asfush. Yesterday I defeated the only brother of the daughter 01 my grandmother' Whom did Ramesh defeat?

a) Son

c) Brother e) COUSin

b) Father

d) Father-In-law

12

When Amir saw ManJeet he recalled He IS the son of the father of the mett-er of my daughter' Who IS Manjeet ?

a) Brother-in-law cj Cousin

e) Nephes

b) Brother d) Uncle

13

Introducing a man a woman said He rs the only son of my mother's mother How IS the woman related to the man?

a) Mother b) Aunt

c) Sister

e) None of these

d) Niece

14

POinting to a man In a photograph Asha said 'HIS mother's only daughter IS my mother' How is Asha related to that man?

a) Nephew c) Wife

e) Grand-daughter

b) SIster d) Niece

53

15 Showing the lady In the park, Vineet sa.c, She IS the daughter of my grandfather's only son' How is Vineet related to that lady?

a) Brother c) Father

e) None of these

b) Cousin d) Uncle

16

tntr odc cir«, a man, a woman said, 'His wife is the or~ly daughter of my father' How was that man related to the woman?

a) Brother

c) Maternal Uncle e) None of these

b) Father-in-law d) Husband

17 Oeepak said to Nitm. That boy playing with the football IS the younger of the two brothers of the daughter of my father's wife' How is the boy playing football related to Oeepak ?

a) Son

c) Cousin

e) Brother-in-law

b) Brother d) Nephew

18

Pointing to the lady on the platform, Manju said, 'She is the sisler of the father of my mother's son' Who is the lady to ManJu ?

a) Mother c) Aunt

e) None of these

b) Sister d) Niece

19 Amtt said 'This girl is the wife of the grandson of my mother' Who is Ami! to the girl?

a) Father c) Husband

e) None of these

b) Grandfather d) Father-in-law

20 POinting to a man In a photograph, a woman said 'HIS brother's father is the only son of my grandfather' How IS the woman related to the man in the photograph?

a) Mother c) Sister

e) Grandmother

54

b) Aunt

d) Daughter

21 Showing the man receiving ~f')e prze 8eema said, 'He is the brother of my uncle's daughter' Who is the man to Seema ?

a) Son

b) Brother-in-law

c) Nephew e) Cousin

d) Uncle

22 Painting to a person a man said to a woman, 'HIS mother is the only daughter of your father,' How was the woman related 10 the person?

a) Aunt c) Wife

e) None of these

b) Mother d) Daughter

23 Introducing a girl, Vipin said, "Her mother is the only daughter of my mother-in-law" How is Vipin related to the grrl ?

a) Uncle c) Brother

e) None of these

b) Husband d) Father

24

Pointing to a lady, a man said, 'The son of her only brother is the brother of my wife' How is the lady related to the man?

a) Mother's Sister

b) Grand mother

c) Mother-in-law

d) Sister of Father-in-law

e) Maternal Aunt

25, Pointing to an old man, Kailash said, 'His son is my son's uncle How is the old man related to Kailash ?

a) Brother

b) Uncle

c) Father

d) Grandfather

e) None of these

DIRECTION SENSE TEST

In this test, the questions consist ofa sort of direction puzzle. A successive follow up of directions is formulated and the candiate is required to ascertain the final direction or the dhancc between two points. The test is meant to judge the candidate's ability to

trace and follow correctly and scns the direction correctly.

I ILLUSTRATIVE EXAMPLES I

N*N NE

W E

SW SE

S

The adloining figure shows the four main directions (North N. South S, East 1', West W) and four cardinal directions (North East NE, Nowrth West NW, South West SW, South East SE) to help the candidates know the directions.

Ex.1.A man is facing west. He turns 45() in the clockwise direction then another 1 ROn in the same direction and then 270IJ in the anticlock wise direction. Which direction is he facing now?

a) South b) North-West

c) West

d) South- West B,.,

"~OO

A~U!~

Sol. Clearly. the man initially faces in the direction OA. On

moving 4Y) clokwi sc, he faces in the direction 01'3. On further

moving 1800 clockwise. he faces in the direction ~C. Finally.

on moving 2700 ant ic lockwise, he faces in the direction 00.

which is South- West. J fence. the answer is (d).

Ex.2.0nc day. Ravi left home and cycled 10 km southwards. turned right and cycled 5 km and turned right and cyc lcsd 10 km and turned left and cycled 10 km. Ilow many

~:,~;:,~~~l:,;:j;,~~f'~;;';:~:~~~ ~~~;i~~:':::~ ::m j - - ,- -1 io

light again and moves 10 km upto D and j inal ly turns left and

moves 10 kill upto I C

Thus. his ditancc from initial position A

~ AI' ~ AD + DE

5km

B

~ Be + DE ~ (5 + 1 O)km ~ 15 km.

Hence, the answer is (b),

Ex.3.A child is looking for his father. He went 90 metres in the cast before turning to his right. He went 20 metres before turning to his right again to look for his father at his uncle's place 30 metres from this point. His father was not there. From

55

there. he went 100 metres to his north before meeting his father in a street. How far did the son meet his father from starting point?

a) 80 metres b) 100 metres c) 140 metres

Sol. Clearly. the child moves from A 90 m eastwards upto B, then turns right and moves 20 m upto C, then turns right and moves 30 m upto D. Finally. he turns right and moves 100m

upto E.

Clearly. AB ~ 90m. BF ~ CD ~ 30 m.

SO. AI' = AS - SF = 60 m.

Also. DE = 100 m, DF = BC = 20 m

So. EF = DE - OF = 80 m.

A

d) 260 metres

, E

~:A

30m C

F

o

lIis distance from starting point A = AE =,[AF'+EP---

~ (60)' + (80)'

~3600 + 6400 = Jl 0000 = 100 m.

Hence. the answer is (b).

Ex.4.Kailash faces towards north. Turning to his right, he walks 25 metres. He then turns to his left and walks 30 metres. Next, he moves 25 metres to his right. He then turns to his right again and walks 55 metres. Finally, he turns to the right and moves 40 metres. In which direction is he now from his starting point?

a) South-west b) South c) North-west d) South-west

Sol. Kailash turns towards right from north direction. So, he

walks 25m towards eat upto B, turns left and moves 30 m upto C, turns right and goes 25 m upto D. At D. he turns to right towards the south and walks 55m upto E. Next,

he again turns to right and wal sk 40 m upto F, which is his

final position. F is to the south-east of A. So, he is to the

25m

25m

J{)m

4 m

south-east from his starting point. Hence, the answer is (d).

Ex5.Deepa moved a distance of75 metres towards the north. She then turned to the left and walking fOT about 25 metres, turned left again and walked 80 metres. Finally. she turned to the right at an angle of 45°. In which direction was she

moving finally ?

a) North-east b) North-westc) South-east d) South-west

Sol. Deepa started from A, moved 75 m upto B, turned left and

walked 25 m upto C. She then turned left again and moved 80 m upto D. Turning to the right at an angle of 45°. she was finally moving in the direction DE i.e., South-West.

Hence. the answer is (e).

56

o

25m

7S m

80m

45°,

E

Ex.6.Kunal walks 10 km towards North. From there he walks 6 km towards South.

Then. he walks 3 km towards East. How far and in which direction is he with

reference to his starting point?

a) 5 krn West b) 7 km West c) 7 km East d) 5 km North-west

Sol. Clearly. Kunal moves from A 10 km northwards upto B, B then moves 6 km southwards upto C, turns towards East and walks 3 krn upto D.

Then. AC ~ (A.B - BC) ~ (10 - 6) ~ 4 krn:

CD = 3 krn.

So, Kunal 's distance from starting point A

= AD =J AC' + CD' ~J'4i+3i = 5 km.

Also. D is to the North-east of A. Hence. the answere is (d).

Ex.7 .. Johnson left for his office in his car. He drove 15 km towards north and then 10

10 km 1-__:;3...:k~m.:.:..._ 0

c

6km

A

Sol.

km towards west. He then turned to the south and covered 5 km. Further. he turned to the east and moved 8 km. Finally, he turned right and drove 10 km. How far and in which direction is he from his starting point?

a) 2 km West b) 5 km East c) 3 km North

d) 6 km South e) None of these C~'Okm •

Clearly. Johnson drove 15 km from A to B northwards and ."'"

then 1 0 km from B to C towards west. He then moves 5

km southwards from C to D and 8 km eastwards upto E, "m E ""

Finally. he turned right and moved 10 km upto F .

. '. A and F like in the same straight line and F lies to the 10 km

west of A.

So. johnson' s distance from the starting point A = AF = (BC - DE) = (10 - 8)km = 2 km. Hence, the answer is (a).

Ramana starts walking towards West After walking 10 metres, he turns towards North After walking 20 metres, he turns towards East and walks 10 metres. How far IS he from his original position and in which direction?

1) 20 metres. North 2} 10 metres, North

3) 10 metres, South 4) 20 metres, South

Vijayan started walking towards South. After walking 15 metres he turned to the left and walked 15 metres. He again turned to his left and walked 15 metres. How far is he from his original position and in which direction?

1} 15 metres, North 2} 15 metres, South

3) 30 metres. East 4) 15 metres. West

5) None of these

Suman IS 40 metres South-West of Ashok Prakash IS 40 metres South-East of Ashok Prakash IS in which direction cr Suman

1) South 2) West 3) East

4'1 North-East 5) None of these

Arun started walking from a point 'P' towards North and walked 30 metres. He then turned left and walked 40 metres and reached 'Q' At what minimum straight line distance IS the point 'Q' from 'P: and In which direction?

1) 50 metres North-East

2) 60 metres North-West

3) 50 metres North-West

4) 80 metres North-West

5) None of these

Starting from a point 'M' Harish walked 18 metres towards South He turned to tus left and walked 25 metres He then turned to hrs left and walked 18 metres. He again turned to his left and walked 35 metres and reached a point t 'P How far Hanan is from the point 'M' and in which direction?

1) 10 metres East 2) 10 metres West

3) 35 metres West 4) 10 metres South

5) None of these

Starting from a pcmt Raju walked 12 metres towards North. then he turned fight and walked 10 metres, he again turned right and walked 12 metres, then he turned lef: and walked 5 metres How far is he now and in which direction from the starting point?

1) 27 metres towards East

2) 5 metres towards East

3) 10 metres towards West

4) 15 metres towards East

5) None of these

58

7. Facing towards South, Pramod walked 50 metres. He then turned to his right and walked 30 metres. He again turned right and walked 50 metres. How far is he from his original position and towards which direction?

1) 20 metres East 2) 30 metres South

3) 20 metres North 4) 30 metres West

5) None of these

Starting from a point 'P' Satish walked 20 metres towards South. He then turned left and walked 30 metres. He then turned left and walked 20 metres. He again turned left and walked 40 metres and reached a point Q. Howfar andm which direction IS the point Q from the point P?

1) 20 metres West 2) 10 metres East

3) 10 metres West 4) 10 metres North

5) None of these

A man walks 30 metres towards South, then turning to his right he walks 30 metres Then turning to his left he walks 20 metres Again turning to his left he walks 30 metres How far is he from his starting position?

1) 30 metres 2) 20 metres

3) 80 metres 4) 60 metres

5) None of these

10 Somu and Kalu together walk for 15 metres then Kalu turns left and Somu turns fight and they continue to walk for 10 metres. Kalu stops there and Somu continues to walk for another 10 metres Haw much has Somu walked?

1) 45 metres 2) 35 metres

3) 25 metres

4) Cannot be determined

5) None of these

11 Facing towards South Ram started walking and turned left after walking 30 metres. he walked 25 metres and turned left and walked 30 metres How far is he from his starting position and In which direction?

1) At the starting point only

2) 25 metres, West 3) 25 metres, East

4) 30 metres, East 5) None of these

12 A man walks 10 Kilometres towards North.

From there he walks 6 kilometres towards South. Then he walks 3 kilometres tawards East. How far and In which direction is he With reference to hiS starting point ?

1) 7 kilometres towards East

2) 5 kilometres towards West

3) 5 kilometres towards North-West

4) 7 kilometres towards West

5) None of these

13 Rakesh is standing at a polnt He walks 20 metres towards East and further 10 metres towards South He then waks 35 metres towards the West and further 5 metres towards East. What is the straight distance In metres between his starting point and the point where he reached last?

1)0 2)5 3)10

4) Cannot be determined

5) None of these

14 Ashok started walking towards North After walking 30 metres he turned left and walked 40 metres. He then turned left and walked 30 metres He again turned left and walked 50 metres How far is he from his onqinal position ?

1) 50 metres 2) 40 metres

3) 30 metres 4) 20 metres

5) None of these

15 FaCing towards North, Ravi walked 30 metres He then turns left and walks 50 metres He again turns left and walks 30 metres How far is he from his original position and towards which direction?

1) 30 metres North 2) 20 metres East

3) 20 metres West 4) 20 metres South

5) None of these

16 Suman walks 15 metres towards West, then turns to her fight and walks 15 metres and then turns to her left and walks 10 metres. Again turning to her left she walks for 15 metres What is the shortest distance between her starting point and her present position?

1) 55 metres 2) 30 metres

3) 45 metres 4) 25 metres

5) 15 metres

17 Facing towards North, Gopal walks 20 metres, then he turns left and walks 40 metres He again turns left and walks 20 metres then he turns right and walks 20 metres How far is he now from his original position?

1) 20 metres

2) 30 metres

3) 60 metres

4) 50 metres

5) None of these

18 A man goes towards east 5 kilometres, then he takes a turn to soutn-west and goes 5 kil.ometres. He again takes a turn towards North-West and goes 5 kilometres With respect to the point from where he started, where is he now?

1) At the starting point

2) In the West

3) In the East 4) In the North-West

5) In the South-West

19 Ashok is standing at P He walks 10 metres towards South: then he walks 20 metres towards West: then he walks 10 metres towards South; then he walks 20 metres towards East and then he walks 5 metres towards North and reaches Q What IS the straight distance between P and Q In metres?

1)0 2)5 3)10

4) 15 5) Noneolthese

20 Vljay starts walking straight towards East. After walking 75 metres he turns to the left and walks 25 metres straight. Again he turns to left, walks a distance of 40 metres straight, again he turns to left and walks a distance of 25 metres How far is he from the starting paint?

1) 140 metres 2) 50 metres

3) 115 metres 4) 25 metres

5) None of these

21 A walks 10 metres In front and 10 metres to the right Then every time turning to his left he walks 5, 15 and 15 metres respectively How far ·5 he now from tns starting point?

1) 5 metres 2) 10 metres

3) 15 metres 4) 20 metres

5) Cannot be determined

22 Facing towards North, Ravr walked 30 metres He then turns left and walks 50 metres He again turns left and walks 30 metres How far is he from his original position and towards which direction?

1) 50 metres West 2) 50 metres East

3) 30 metres East 4) 30 metres West

5) None of these

23 Mohan moves 20 metres East and then turns to his left and then moves 15 metres and then he turns to his right and moves 2S metres. After this he turns to his right ahd moves 15 metres Now how far is he from his starting point?

1) 0 metres 2) 40 metres

3) 50 metres 4) 25 metres

5) 45 metres

24. Kishan was facing East He walked 4 kms forward and then after turning to his right walked 3 kms Again he turned to his right and walked 4 kms. After this he turned back Which direction was he facing at that lime?

1) East 2) West 3) North

4) South 5) North-East

25 Milind goes 30 metres North, then turns fight and walks 40 metres, then again turns right and walks 20 metres, then again turns right and walks 40 metres. How many metres is he from his original position?

1) 0 2) 10 3)20

4)) 40 5) None of these

59

Oeepak starts walking straight towards east After walking 75 metres, he turns to the left and walks 25 metres straight, Again he turns to the left, walks a distance of 40 metres straight again he turns to the left and walks a distance of 25 metres How far is he from the

straight point? a) 140 metres c) 115 metres

e) None of these

b) 50 metres d) 25 metres

Kishenkant walks 10 kilometres towards North From there, he walks 6 kilometres towards South Then, he walks 3 kilometres towards East. How far and in which direction is he with reference to his starting point?

a) 7 kilometres East .

b) 5 kilometres West

c) 5 ktlornetr es North-East d:, 7 kilometres West

A man leaves for hts office from nis house He walks towards East After moving a distance of 20 m. he turns towards South and walks 10 m. Then he walks 35 m towards the west and further 5 m towards the north. He then turns towards east and walks 15 m What is the straight distance in metres between his initial and final positions?

a)O ui s e)10

d) None of these

e) Cannot be determined

Gaurav walks 20 metres towards North He then turns left and walks 40 metres He again turns left and walks 20 metres Further he moves 20 metres after turning to the right How far IS he from tus original position?

a) 20 metres b) 30 metres

c) 50 metres d) 60 metres

e) NOlle of these

Ankit started walking towards North After walking 30 metres, he turned left and walked 40 metres He then turned left and walked 30 metres lie again turned left and walked 50 metres How far was he from his original position?

a) 50 metres c) 30 metres

e) None of these

b) 40 metres d) 20 metres

Gopal starts from hiS house towards West After walking a distance of 30 metres, he turned to wards right and walked 20 metres He then turned left and moving a distance of

60

10 metres. turned to his lefl again and walked 40 metres. He now turns to the left and walks 5 metres. Finally he turns to his left In which direction is he walking now?

a) North b) South c) East

d) soutn-west e) West

A girl leaves from her home She first walks 30 metres In North-West direction and then 30 metres In South-west direction, Next, she walks 30 metres in South-East direction Finally, she turns towards her house. In wllich direction is she moving?

a) North-East b) North-West

c) South-East d) South-West

e) None of these

Sanjeev walks 10 metres towards tne south Turning to the left. he walks 20 metres and then moves to his right After moving a distance of 20 metres, he turns to the right and walks 20 metres, Finally, he turns to the right and moves a distance of 10 metres. How far and in which direction IS he from the starting point?

a) 10 metres North b) 20 metres South

c) 20 metres North d) 10 metres South

e) None of these

Keshav goes 30 metres North then turns right and walks 40 metres, then agai"l turns right and walks 20 metres, then aqain turns ,ighl and walks 40 metres How many metres is he from his Original position?

a) 0 b) 10 c) 20

d) 40 e) NOne of these

10 A man walks 30 metres towards South Then, turning to his right. he walks 30 metres Then, turning to his left, he walks 20 metres, Again, he turns to his left and walks 30 metres How far is he from his initial position?

a) 30 metres b) 20 metres

c) 80 metres d) 60 metres

e) None of these

11 Jitendra starts walking towards East After moving a distance of 20 metres, he turns to nte left and walks 15 metres Then, he turns to his right and moves 25 metres, Next, he turns to his right again and walks 15 metres. How far in metres is he from his starting pomt ?

a) 0 b) 25 c) 40

dj 45 e) 50

12 Starting f'OIT, a point P, Sactun watked 20 metres towards South He turned left and walked 30 metres He then turned left and walked 20 metres He again turned left and walked 40 metres and reached a point Q, How far and In wh-en direction is the point Q from the point P ?

a) 20 metres west b] 10 metres east

c) 10 metres 'Nest d) 10 metres north

e) None ot these

13 Rarnakant walks northwards After a while, he turns ,J 'us I-Ight and a little further to tns left Finally. after walking a distance of one kilometre, he turns to tus left again In which direction IS he rucvmq now?

a) North bl South C) East d) West

14 Joseph started walk.nq towards East After moving a distance of 1 km h e tur n ed southwards and walked 5 kill Again. he turned to East and watked 2 km Finally, he turns to the North and walked 9 km How far IS he from n.s

J) km C) 4 km

d) 3 km e) None of these

15

FaCing towar ds the South, Mahesh started walking and turned left after watcinq 30 metres He then walked 25 metres and turned left and moved (1 distance of 30 metres How far IS he from his starting position and which direction

a) At the starting p0111! bi 2S metres W8St c) 25 metres East d) 30 metres East

e) None these

16 Raj traveus-d fr orn d distance (If 60 metres

to Y at a

walked 70 n-enes walked 50 metr es point -)

20 metres

c) 70 metres

e) None of these

i:J) 50 metres d:1 10 metres

17 Lakshman went 15 kms to the west from my house, then turned left and walked 20 kms He then tur oeo east and walked 25 kms and finally turning left covered 20 kms How far was he from my house?

a) 5 krns b) 10 kms c) 40 kmsd) 80 kms

18 Varun travelled? km eastward, then turned left and travelled 5 km Next. he turned left and travelled? km How far was he from the starting point ?

a) 19 km b) 12 km c) 7 km d) 5 km

19 From his house Lokesh went 15 krns to the North Then he turned west and covered 10 kms Then, he turned South and covered 5 kms. Finally. turning to East. he covered 10 kms In which direction IS he from hts house?

a) East b) West c) North d:1 South

20 GOing 50 m to the South of her house. Radtuka turns left and goes another 20 m Then, turning to the North, she goes 30 m anc then starts walking to her house In which direction is she walking now?

8) North-West b) Norlh

c) South-East d) East

21 A walks 10 metres in front and 10 metres to the right Then every time turning to his left he walks 5, 15 and 15 metres respectvely How far is he now from tus starting pomt ?

a} 10 metres b) 20 metres

c) 5 metres d) 15 metres

e) 23 metres

22 Rekha who IS faCing South turns to her left and walks 15 metres, then she turns to her teft and walks 7 metres then tacmq West she walks 15 metres How far is she t-orn her

original position? a} 22 metres

c) 4t metres

e) None of these

bl 37 metres d) 7 metres

23

Dtlip walks 20 metres North Then lie turns fight and walks 30 metres Now he turns right and walks 35 metres. Now, turning left he

walks 15 metres tUf'l,ng left he aqam

walks 15 metres In direction and how

far is he from his original position?

a) 15 metres East bi 45 metres East

c) 15 metres West d) 45 metres West

e: None of these

24 Wilson was gomg home from lus school He ursr moved 3 km southwards and then turned to lus left, moving 2 km From there, he moved to rus left and walked '<, km Further. he turned to lHS left agair and moved 1 km to reach tus house In which direction was.his (louse from t.rs school?

a) North b) Souih c) East

d) West e) North-West

25 The door of Aditva's house faces the East From the backside of the house. he walks straight 50 metres. then turns to the fight and walks 50 metres again Finally, he turns towards left and Slops after walking 25 metres Now, Aditya is In which direction from the starting point?

a) South- East b) North-East

c) South-West d) North-West

_61

PUZZLE TEST

This section comprises of questions put in the form of puzzles involving certain number of itrncs. be it persons or things. the candidate is required to analyse the given information. condense it is a suitable form and answer the questions asked.

The questions on Puzzle Test may be of any of the following types:

I. Classification Type Questions II. Seating I Placing Arrangements III. Comparison Type Questions IV, Sequential order of things

v. Selection based on given conditions

VI. Questions invloving family members - their relationship, their professions. their preferences etc.

VIl.Jumbled up Problems

TYPE I : CLASSIFICATION TYPE QUESTIONS

This type consists of questions in which certain items belonging to different groups or possessing different qualities are given along with some clues with the help of which the candidate is required to group and analyse the given items and answer the questions accordingly.

Ex.l. Read the following information carefully and answer the questions that follow:

There arc six cities A, B. C, D, E and F. A is not a hill station.

Band E are not historical places. D is not an industrial city.

A and D are not historical cities. A and B are not alike.

I . Which two cities are industrial cnetres ?

a) A and B

b) E and F

c) C and D d) Band F e) A and D

2. Which two cities are historical places?

a) A and C b) E and F c) C and F d) Band E e) A and D

3. Which two cities are hill stations

a)AandB

b) C and A c) Band D d) A and F e) None of these

4. Which city is a hill station and an industrial centre but not a historical place?

a) E b) F c) A d) B e) C

5. Which two cities are neither historical places nor industrial centres?

a) A and B b) D and E c) F and C d) Band D e) None of these

62

Solution: The given information can be analysed as follows:
A B C D E F
Historical x v' x x v'
Industrial v' v' v' ./
Hill Stations x ./ v' ./ v' ./ Since A and B are not alike and becuase A is industrial, B cnnot be industrial but only a hill station, So. we put a cross for B across Industrial.

I. Clearly, A, C, E and F are Industrial Centres. So, the answer is (b).

2. Clearly, C and F are Historical places. So, the answer is (c).1

3. Clearly, B. C, D, E and F are Hill stations. So, the answer is (e).

4. E alone is a Hill station and an Industrial centre but not 8 Historical place So. the answer is (8).

5. Band 0 are neither Historical places nor Industrial centres. So. the answer is (d).

Ex.2 Read the following information carefully and answer the questions that follow :

(i) Five friends P, Q, Sand T travelled to five different cities ofchcnnai. Calcutta.

Delhi, Bangalore and Hyderabad by five different modes of transport ofHus. Train, Aeroplane, Car and Boat from Mumbai.

(ii) The person who travelled to Delhi and not travel hv boat.

(iii)R v.. .. en: to Bangalorc by car and Q went to Calcutta by acroplancc. (iv)S travelled by boat whereas T travelled by train.

(v} Mumbai is not connected by bus to Delhi and Chennai.

Which of the following combinations of person and mode is not correct '!

a) P-l3us b) Q- Aeroplane e) R - Car

d) S - Boat e) T ._-- Aeroplane

2. Which of the following combinations is true for S ?

a) Delhi-Bus b) Chennai -Bus c) Chennai --Boat

d) Data inadequate

e) None of these

3. Which of the following combinations of place and mode is not correct?

a) Delhi -v-Bus b) Calcutta -Aeroplane c) Bangalore -C'1r

d) Chennai -Boat e) Hyderabad -Bus

4. The person travelling to Delhi went by which of the following modes ?

a) Bus b) Train e) Aeroplane d) Car e) Boat

5. Who among the following travelled to Delhi?

a) R b) S e) T d) Data inadequate e) None of these

63

Solution: The given information can be analysed as follows:

(AIMode of Transport : R travels by Car, Q by Aeroplane, S by Boat and T by Train. Now, only P remains. So, P travels by Bus.

(Hj Plncc of Travel : R goes to Bangalorc, Q to Calcutta. Now, bus transport is not available for Delhi or Chcnnai, So, P who travels by Bus goes to lIyderaqbad. S travelss by boat and hence, by (ii), did not go to Delhi. So, S goes to Chcnnai. Now, only T remains. So, T goes to Oelhi.

TYPE 2: SEATING / PLACING ARRANGEMENTS

In this type of questions. some clues regarding seating or placing sequence (linear or circular) of some persons or items is given. The candidate is required to form the proper sequence using these clues and answer the questions accordingly.

Ex. J. Read the following information carefully and answer the questions given below:

Six persons A. B, C, D. E and F are sitting in two rows, three in each. E is nut at the end of any row.

[) is second to the left ofF.

C. the neighbour of E, is sitting diagonally opposite to D. B is the neighbour of F.

1. Which of the following arc sitting diagonaly opposite to each other?

a l l+and C b)DandA c)Aande d)AandF

c) A and B

2. Who is facing B ?

a) A b) C c) 0 d) E c) F

3. Which of the following are in the same row?

a)AandE b)EandD c)C.ndB d) Aand B

c) C and E

4. Which of the following arc in one of the two rows?

a) FBC b) CEB

c)DBF

d)AEF

e)-ABF

5. After interchanging seat with E, who will be the neighbours of D in the new position?

alC and Ab) F and B c) Only B

d) Only A

e) Only C

Solution: The given information can be analysed as follows:

E is not at end. So, E must be in the middle of one of the rows. D is second to the left ofF. So, order of the row must be 0 - F.

C is neighbour of'E and is sitting diagonally opposite to D means C is under Fin

the other row i.e., 0 - F - E C

B is the neighbour of F.

So, the arrangement must be D B F AE C

64

1. Other than D and C (given), A and F arc sitting diagonally opposite to each other. as seen in the arrangement. So, the answer is (d)

2. Clearly, E is opposite to B in the other row. So, E is facing B and the answer is (d) .

. '. Clearly. from amongst the given alternatives, A and E arc in the same row, So, the answer is (a)

4. Clearly, from amongst the given alternatives, D. Band F are in the same 1"0\.\'. So. the answer is (c)

5. Clearly, neighbours ofE and A and C. So. on interchanging the seat with E. the new neighbours ofD will be A and C. So. the answer is (a)

Ex.2.Eight hooks are kept one over the other. Counting from the top. the second. fifth and sixth books arc on Plays. Two hooks on Plays are between two hooks on Composition. One book of Plays is between two books on Poetry while the hook at the top of the book ofLiterature is a book of Composition. Which book is fourth from the top '?

a) Plays b) Poetry e) Composition d) Literature

Solution: We analyse the given information as follows:

Let C denote 'Composition', P denote 'Plays'. Po denote 'Poetry' and L denote 'Literature' .

4 6 7
P P I'
C p p C
Po P Po
C L
So. the arrangement becomes:
2 4 5 7
Po r Po C P P C L Clearly, the fourth book from the top is on Composition. So, the answer is (c).

Ex.3.Read the following information and answer the questions that follow:

(i) Six friends A. B, C, D, E and F are sitting in a closed circle facing the centre. (ii) E is to the left ofD.

(iii)C is between A and B.

(iv)F is between E and A.

1. Who is to the left ofB ?

a) A b) C e) D d) E e) None of these

2. Who is to the right ofC ?

aj A b)B c)D d)E elF

65

3.

Which of the above given statements is superf'luous ?

a) (i) h) (ii) c)(iii) d) (iv)

e) None Ofth:o'C

B E

Solution: Clearly. in the c irle the arrangement is as shown

I. (c):Clcarly. D is to the left of'B. .

1. (a):Clcariy. A is to the right ofC. . A F (cj.Since all the statements are necessary to determine the arrangement, none of

them is superfluous.

TYPE 3 : COMPARISON TYPE QUESTIONS

In such tupe of questions. clues are given regarding comparisons among a set of persons or things with respect to one or more qualities. The cadidate is required to analyse he whole information. form a proper ascending / descending scquenccc and then answer the given questions accordingly.

Ex.t.Read the followig information and answer the qucstionsgiven below it:

There are five friends - Sachin. Kunal. Mohit, Anuj and Rohan. Sachin is shorter than Kunal but taller than Rohan.

Mohit is the tallest.

Auuj is a little shorter than Kunal and little taller than Sachin. Who is the shortest?

a) Rohan 0) Suchin

c) Anuj

d) Kunal

e) None of these

I f they stand in the order of their heights. who wi II be in the middle?

a) Kunal b) Rohan c) Sachin d) Anuj e) None of these If they stand in the order of increasing heights. who will be the second? a) Anuj 0) Sachin c) Rohan d) Kunal e) None of these Who is the second tallest '?

4.

a) Sachin h) Kunal c) Anuj d) Rohan

5. Who is taller han Anuj but shorter than Mohit ?

0) None of these

a) Kunal b) Rohan c) None of these

Solution: Let us denote the five boys by the first letter of thier names, namely S, K, M_ A and R.

c) Sachin

d) Data inadequate

Then, R < S < K< M and S < A < K.

R<S<A<K<M

1.(a): Rohan is shortest.

2.(d): Anuj is in the middle.

3.(b): In the order of increasing heights i.e. shortest to tallest, Sachin is second.

4.(b): Kunal is second tallest.

5.(a): Kunal is taller than Anuj but shorter than Mohit.

66

Ex.2. Read the information giveen below and answer the questions that follow (i] There is a group of five girls.

(ii) Kamini is second in height but younger than Rupa, (iii)Pooju is taller than Monika but younger in age.

(ivrkupa and Monika are of the same age but Rupa is tallest between them. (v) Neelam is taller than Pooja and elder to Rupa.

Tfthey are arranged in the ascending order of height. who will be in third position?

a) Monikab) Rupa c ) Monika or Rupa d) Data inadequate e) None

Ifthey are arranged in the desceending order of their ages. who will he in fourth

position?

a) Monika or Rupa c) None of these

3. To answer the question "who is the youngest person in the group". which of the

b) Kamini

0) Monika

d) Data inudcquute

given statements is superfluous?

0) Only (i)

d) Either (il or (iv)

b)Only(ii)

c) None of these

c) Only (v)

Solution: We first find the sequence of heights :

By (iii). we have: M < P. By (v), we have: P< N.

Now. Rupa is tallest and Kamini is second in height. So, the sequence of heights is : M < P < N < K < R. Now, we determine the age sequence;

By (ii). we have: K < R.

By (iii). we have: P < M.

By (iv). we hay" : R ~ M.

By (v). we have: R <N.

So, the sequence of ages is : N < R = M < K < P or N < R = M < P < K.

I. Clearly. in the increasing order of height, neeJam is in third position.

Hence, the answer is (e).

2. Clearly, in the descending order of agess, Neelam will be in fourth position (because Monika and Rupa both lie at third position).

Hence. the answer is (e).

3. Clearly, konly statement (i) is not necessary.

Hence, the answer is (a)

67

TYPE : FAMILY - BASED PROBLEMS

In such type of questions. some clues are given regarding relationship among different members ofa family and their professions, qualities, dresses, preferences etc. The candidate is required toanalyse the whole information and then answer the given questions accordingly.

Ex. Read the following information carcefully and answer the questions given below it :

0) There is a group of six pI:fSOnS A, B, C, D. E and F from a family. They arc

Psychologist. Munuger, Lawyer, Jeweller, Doctor and Engineer. {ii} 'I'IH:: doctor is the grandfather of'F who is a Psychologist. (iii)The Manager D is married to A.

(iv)C, the Jeweller is married to the Lawyer. (v) B is the mother of F and E.

(vi)There are two married couples in the family.

1. What is the profession of E?

a) Doctor 0) Jeweller c) Manager d) Psychologist

e) None of these

2. 1-10 v, ' is A related to E?

a) Hrother b) Uncle e) None ofthese

_'. How many male members arc there in the family?

a) One b) Three c) Four d) Data inadequate

c} Father

d) Grandfather

c) Cannot be determined

4. What is the profession of A ?

a) Doctor h) Lawyer e) Jeweller d) Manager

e) None of these

:=;. Which of the following is one of the pairs of COlJPh.:S in the family

a) AB 0) AC c) AD d ) Cannot be determined

e) None of these

Solution: (liven F is a psychologist.

A is the mother of F and E means E is the brother or sister of F.

There arcc only two married couples in the family. Since D i:\ married to A. so c. the jeweller. who is married to a lawyer, will be married to B.

Again, the Manager D is married to A means A is the doctor and Grandfather of' l-

and E. Also. no one else is an Engineer. So, the answer is (e)

I. Clearly, E is an Engineer. So. the answe is (e)

2. Clearly, A is the grandfather of'F and E is the brother or sister of F. So, A is the grandfather of'E, Hence, the answer is (d)

3. Since nothing is mentoined about E and F, so the number ofmaled cannot be

determined. Hence, the answeris (e)

68

4. Clearly, A who is the grandfather of D is the doctor. Hence, the answer is (a)

5. Clearly. D. the manager is married to A. So. AD is one of the couples in the

family, lienee, the answer is (c)

TYPE: SELECTION BASED ON GIVEN CONDITIONS

In such type of questions. a few essential criteria for selection of a group or items arc given. The candidate has to keep these conditions in mind and make (he requied selection as per the directions given in each question,

Ex.I. Stud-. the following information carefully and answer the questions given below it :

From amongst Six. boys A. B. C, D. E and F and five girls P. Q, R. Sand T. a team of six is tobe selected under the floowing conditins :

(i) A and D have to be together.

(ii) C cannot go with S.

(iii) S and T have to be together. (iv) B cannot to teamed with E (v) B cannot go with P.

(vi) Band R have to be together. (vii) C and Q have to be together.

I. If there be five boys in the team, the lone girl merner is:

alP b)Q c)R ai s

2. Ifincluding P. the team has three girls. the mcbers are:

a) BCF()R b) ADEST c) ADHST d) BFRST

3. If the team including C consists off OUT boys, the members ofthe learn other than C arc:

o)ADEI'Q

b)ABDQR c)DEFAQ d)BEFRQ

4. Iffour members including E have to be boys. the members other than E arc:

a) ABCQR h) ADFST c) BCFQR d) ACDFQ

5. Jffour members have to be girls, the members of the team are:

aJ BCPQRS

h) BFPRST c) BCQRST d) BCPQRT

Solution: The given questions may be handled as under:

1, I n a team of six if fi ve boys are robe selected then both A and D together are selected, IfC is sleeted, a girl Q will be selected.

From Band E, one has to be selected. So, we select E because B will he accompanied by a girl. The fifth boy will be F. So, (he only girl will be Q in the team A C D EO F Q. Hence, the answer is (b)

2, IfP is included, D and hence a cannot be included. If Q is selected, C has tobc selected and so S cannot be selected. T goes with S. So, T is also not selected. The third girl can be R, With R, B "ill be selected but with B, E cannot be selected. So, the sixth member can be F only. i.e., the team be-comes P Q C R B F. So, the answer is (a)

69

3. If tea", contains C,Q will be included. If another girl included is R, B will be there and hence E cannot be there. A and D have to be together. So, they are also included and only F can be excluded. Thus, the team is C Q! R BAD. So, the answer is (b).

4. If E is included, B cannot be included. A and D have to be together. SO,m they are both included. Without B. R will not be theree. With D, P cannot be theree. So. two girls together can be only Sand T. Irs is there, C cannot be there. So the fourth boy can be F alone. Thus, the team becomes E ADS T F. So, the answer is (b).

5. In four girls, S and I are taken together. With S. C cannot be there. So, Q will not be there. If!' is included, D and hence A cannot be there. IfR is included, B will be theree and hence E cannot be there. So, only F can be there. Thus, the team is S T P R B F. So, the answer if (b).

TYPE: SEQlJENTJAL ORDER OF THINGS

In this type of questions, some clues are given regarding the order of occurrence of certain events. The candidate is requicd to analyse the given information, frame the right sequence and then answer to questions accordingly,

Ex.t . Read the following information and answer the questions given below it:

Six plays - A. B. C. D. E and F are to he staged. one on each day from Monday

to Saturday. The schedule of the play-, is tobe in accordance with the following: {i) A must be staged a day before E.

(ii) C must bot be staged on Tuesday.

(iii) B must be staged on the day following the day on which F is staged.

(IV) D must be staged of friday only and should not be immediately preceded by B. {v] E must not be staged on the last day of the schedule.

Which of the following plays immediately follows B ?

a) A b) C c) D d) E e) F

Which of the following plays is on Monday ?

u ) E b) F c) C d) B c) A

4.

Play D is between ddwhich of the following pairs of plays '!

a)ilandE b)EandF c)AandE d)CandE

e) C and F

Which otthe following is the schedule of plays, with the order oftheir staging from Monday?

a) E,A,B,F,D,C b) A,F,B,E,D,C

c) A,F,B,C,D,E

d) F,A,B,E,D,C

c) None of thesee.

5. Play C cannot definitely be staged on which ofthe following days in additior, ro Tuesday?

a) Monday

b) Wednesday c) Friday

d) Tursday

e) Saturday

70

Solution: Clearly. D must he staged on Friday. A must he staged he force E i.c .. order AE must be followed. But E cannot be staged on last day. Also. B must be staged immediately after F i.e .. order FB must he followed. But B cannot precede D. So. F and B can. he staged on Monday and Tuesday and A and E on Wednesday and I'hur sday. C_ which cannot be staged on Tuesday shall be staged on Saturday. Thus, the order followed will be :

Monday

Tuesday

Wednesday Thursday

friday

Saturday

- .. --- .----~-- --1------+------1--------.--

B A E IJ C

1. Clearly, A immediately follows B. So, the answer if {a}.

2. F will be played on Monday. So, the answer is (b).

3. Play D is between E and C. So, the answer is (d).

4. Clearly, order ofstgging of plays is F, B. A. E. D, C. So the answer is (c).

5. C cannot be staged on Friday as wcl! because D has tohc staged 011 that day.

E):.2.Reali the following information carcfuly and answer the questions given below it: (i) Eight dcctors P,Q. R,S:r.U.V and \)\' visi t a charitable dispensary every da) except on a holiday i.e., Monday.

(ii) Each doctor visits for one hour form Tuesday to Sunday except Saturday.'.

The tImings are Q a.m. to 1 p.m. to 6 p.m. to .2 p.m. is lunch break.

(iii) On Saturday, it is open only in the morning j .e .. <) a.m. to 1 p.m. and each

doctor visits for only half and hour.

(iv) No other dec tor visits the dispensary before doctor Qand after doctor U. {v) Doctor W comes immediately after lunch break and is followed by R.

(vi} S COllies in the same order as P in the afternoon session.

I. Doctor P visits in between which of the following pairs of doctors '?

a)SandV il)UandW c)RandW d)RandlJ

e) None of these.

3.

At what time the visit of doctor R is over on Sunday'!

0) I p.m. b) 3 p.m. c) r p.m, d) 5 p.m. 0) Nonc oftbcsc

At what time the visit of Doctor T would be over on Saturday?

a) 10 a.m. b) I I a.m. c) Either 10 a.m. or II a.m.

d) Data inadequate

e) None of these.

4. If the lunch break and subsequent visiting hours are reduced by 15 minutes, at what time Doctor U is expected to attend the dispensary'?

a) 1. I 5 p.m. h) 4 p.m c ) 4.15 p.m. d) 4.45 p.m. e ) None of these.

71

Solution: WQe first form the sequence of visit using (iv).(v) and (vi).

From (iv), we know that Q visits first and U visits last.

From (v ). we know that W visits first after break and is followed by R. From (vi), we know that P visits after break.

Thus. the sequence of vist after break becomes W R P U.

Also. S has the same position in morning session as P in afternoon session. So, sequence of visit before break is Q. TIV. S. V/T.

I. (d): Clearly. P visits between Rand U.

2. (c): The time of visit ofW is 2 p.m. to 3 p.m .• that o F docto r R is 3 p.m. to 4 p.m.

So. the visit of doctor R is over at 4 p.rn.

3. (cj.Cteerly. T visits either second or fourth. So. the time of visit on Saturday will

be either 9.30 a.m. or 10.30 a.m. Thus, T s visit willbe over at wither 10 a.m. or II a.m.

4. (bj.Cleariy, as mentioned, lunch break will he over and doctor W will visit at

1.45p.m .• doctor R will visit at 2.30 p.m., doctor P will visit a13.15 p.m. and U will visit at a p.m.

72

SEQUENTIAL OUTPUT TRACING

In this type of questions, a message comprising ofrandrnoised words or numbers is given as the input followed by steps of rearrangement to give sequential outputs. The candidate is required to trace out the pattern in the given rearrangement and then determine the desired output step, according as is asked in the questions.

Example: Study the following information to an.fwer the given questions:

A word arrangement machine, when given an input I ine of words. rearranges them following a.putilNlar rule in each step. The following is an Hlustration of input and the steps.cf'rearrangernent :

Input As ifit on an Zoo figure Of in at

Step I an As ifiton Zoo figure Of in at

Step II Step III Stqt IV

as As at if it orr Zoo fi~e:'Ofin an As at figure if it on Zoo-of i1'l an As at figure if in il on ZDf> Of

Stqt V an As at ftgW''' if im il Of on Zoo

(and Step V is thd!asBlep for this Input).

As per the rules followed 'olhe above steps. find out in the given questions the appropriate step for the given inpuL

I. Which of the foll.crwing will be Step Il for the given input?

... pat : am ace all ifls a) ace all am Is. if

d) ace all am ifls

b) all am ace ifls d) None of these

c) Is -ifam ace all

2. topul: you aee at fault on this

Which of the following steps would be - are at fault 00 you this?

a) I b) II c) III d) IV c) V

3. Input: Him and His either or her

Which step will be the last step for this input?

a) I b) II c) III d) IV e) V

4. Step IV was like this - an apple at cot was red on one side.

Which of the following will definitely be the input?

a) was cot red an on at one apple side

b) cot an at apple was red on one side

c) apple at an cot was red on one side

d) Cannot be determined

e) None of these

73

Sol.tion :

Clearly, in the given arrangement, the words have been arranged alphabetically in a sequence, altering the position of only one word in each step.

I. Clearly, we have:

I_t am ace all ifls

Step I St.,p II

So, the answer is (d).

2. I_I you are at fault on this

ace am all ifls ace al.am if I s

Step I Step II Step III

are you at fault on this are at you fault on this are at fault you on this

Step IV are at fault on you this

So, the answer is (d).

3. Input Him and His either or her

Step I and Him His either or bet

Step II Step III

and either Him His or her and either her Him Hisor

Since all the words in the given input have been arranged aplhabeticaUy uptill Step 1II, so it is the last step.

Hence. the answer is (c).

4. Tracing the output steps for each ofthe given inputs, we find that Step IV for (a) is the same as that given in the questions, while in (h) and (c), the desired output occurred at Step III.

Input Step I Step II Step III Step IV

So, the answer is (a),

74

was cot red an on at one apple side an was cot red on atone apple side an apple was cot red on at one side an apple a1 was cot red on one side an apple at cot was red on one side

LOGICAL VENN DIAGRAMS

This section deals with questions which aim at analysing a candidate's ability to relate a certain given group of items and illustrate it diagrammatically.

Here are a few different types of Venn diagrams with their implications made clear.

Suppose you arc given a group of three items. Then,

1. if the items evidently belong to three different groups, the Venn diagram representing it would be as shown alongside.

Ex. Do~, Engineers, Lawyers

Tbesc three items bear no relationship to each other. So, they are represented by 3 disjoint figures as shown in Fig. I.

if one item belongs to the class of the second and the second belongs to the class of third. then the representation is in the form of three concentric circles. as

2.

shown in Fig. 2.

Ex. Se('oDCf!l, Minutes, Hours

Clearly. seconds are a part of minutes and minutes are a part of hours. So, the Venn diagram would be as shown in the adjoining figure with circle A representing Seconds, circle R representing Minutes and circle C representing Hours.

3. if two separate items belong to the class of the third, they me represented by two disjoint circles inside a bigger circle as shown in Fig. 3,

Ex. Table, Chair, Furniture

Clearly, table and chair arc separate items but both are items of furniture. So, they would be represented as in the adjoining figure with circle A representing Table circle B representing Chair and circle C representing Furniture.

4. if two items belong to the class to the class of the third such that some items of each of these two groups arc common in relationship, then they arc represented by two intersecting circles enclosed within a bigger circle.

Ex. Males, Fathers. Brothers

Clearly, some father may be brothers. So. fathers and brothers would be represented by two intersecting circles. Also both fathers and brothers are males. So, the diagrammatic representation would be as shown in Fig. 4, with circle A representing Father, circle B representing Brother and circle C representing Males.

08 8

Fig.!

Fig.2

Fig.3

Fig.4

75

5. if two items are partly related to the third, and are themselves independent of each other they are represented by three intersecting circles in a line.

Ex. Dogs, Pets, Cats

Clearly, some dogs and some cats are pets. But. all the pets are not dogs or cats, Also dogs and cats are not related to each other. So, the given items would be represented as shown in Fig. 5 with circle A representing Dogs, circle B representing Pets and circle C representing Cats.

6. if the three items are partly related to each other, they are represented as shown in adjoining figure.

E •. Clerks, Gever ... ".' Empt..,.""., Educated Persons

Clearly. some clerks may be government employees and some may be educated. Similarly. some government employees may be clerks and some may be educated. Also, some educated persons may be clerks and some may be government employees. So, the given items may be represented as shown in Fig. 6 with three different circles denoting the three classes.

7. if one item belongs to the class of second while third item is entirely different from the two, then they may be represented by the adjoining diagram.

Ex. Engineers, Human Beings, Rats

Clearly. all engineers are human beings. This would be represented by two concentric circles. But the class of rats is entirely different from these two. Thus, these items would be represented as shown in Fig. 7 with circle A representing Engineers. circle B representing Human Beings and circle C representing Rats.

8. if one item belongs to the class of second and the third item is partly related to these two, they are represented as shown alongside.

Ex. Female s, Mothers, Docters

Clearly, all mothers are females. This would be represented by two concentric circles. But, some females and some mothers can be doctors. So. the circle representing doctors would intersect the two concentric circles. Thus, the diagram becomes as shown in Fig. 8 with circle A representing Mothers, circle B representing Females and circle C representing Doctors.

:76

Fig.S

Fig. 6

DIRECTIONS:

a) Mur af and Rame s h play hockey and football

b) Emanual and Rame sh play football and cricket

c) Murali and Rasheed play tennis and hockey

d) Rasheed and Emanual play cricket and tenrus

QUESTIONS:

1) Name the person who plays cricket, tennis and football

1) Emanual 3) Ramesf

7) Rasheed A)Murail

5) none

2) Name the perosn wno plays tennis. cricket and hockey

1) Emanual 2) Rasheed

3) Ramesh 4) Murati 5) none

3\ Name the pe cs an wb o plays foot-ball cricket and hockey

1) Emanuat 2) Rasheed

3} Rarr-esf ~) Mural, 5) none

4) Name the person who play~ tennis hockey and toot-ball

1;, Emanua' 3) Ramesb

Rasheed 41 Murali

5,

Name the nerson who plays hockey, cricket, ten 'lis arlO foot-ban

1) Emanua' Rasheed

3) Rame sn

~) Mur ah

5) none

STATEMENTS:

a) P, 0 and R are Intelligent

b) P S awl T are hardworking C) S, Rand Tare I'"nest

d) P. Q and T are ambitious

QUESTIONS:

6)

Who among tre following is neither intelligent

nor ambmous

1) P 21 Q

3) R

4) S

7)

Who among the fotlowmq IS neither ambitious nor hardworklf19

1)P 2)0 3)R 4)$ 5)T

8)

Who among the following is hardworking, intelligent. and ambitious but not honest 1)P 2)Q 3)R 4)S 5)T

9) Who-among the following IS neither honest nor hardworking but ambitious

1) P 2) Q 3) R 4) S 5) T

10) Who among the Icllowinq IS arntnttous but not Intelligent

1) P 2) Q 3)R 4)S 5)T

Six fnends Dwarak Narayan Raman, Pavan Surya and Raja live in Aqra Deihl Madras, Indore Bombay and Gwafior not In the same orde Raja and Raman do not live in Bombay 01" Gwalior Dwarak and Narayan do nothve In Madras or In Deihl Pavan and Surya do no: live In Indore or in Agra Raja and Surya do not live in Madras or In Deihl Narayan and Pavan do not live In Gwalior or In Bombay Dwar ak and Raman do not live In Indore or In Agra Surya does not live In Gwalior and pavan does not live III Delhi Raja does not live III Indore

QUESTIONS:

11) Raja lives in 1) Agra

4) Indore

2) Bombay 5) Madras

3) Delhi

12) Raman lives in

1) Gwalror 2) Agra

J) Madras4) Delhi

5) none

13) Who lvesm Gwallor Raman

3) Dwarak

2) Raja

4) Narayan

14)

Who lives 10 Bombay

1) Surya 2) Dwarak 3'1 Pavan 4) Raman

15) Pavan lives in 1) Agra

3) Bombay

2) Gwaliur 4) Madras

DIRECTIONS:

5)T

Har! Bhasker. Chand. Suresh and Tukara'u are interested in Hockey, Tennis SWimming, Badmmton and Cricket but not Ir the same order Chand does not play Hockey Han and Tukaram do not play Badminton Hari and Suresh do not play Cricket Tukaram neither plays Hc ckey nor Cricket Bhaskar and Chand are not gOing for SWimming The first letters of the names and the games played by that person is not the same

QUESTIONS:

16) Who plays Tennis

1) Han 2) Bhaskar 3) Chand 4) Suresh

77

171 Wt10 goes for Swimming

1) Bhasker 2) Tukaram

3) Suresh 4} Chand

18:1 Badminton IS played by

1) Han 2) Suresh

3) Chand 4) Tuicaram

191 Suresh plays 1) Hockey 3) Cricket

2) Badminton 4) Swimming

201 Who plays Cricket 1) Bhaskar

3) Tukaram

2) Chand 4) Suresh

DIRECTIONS:

Four persons Han Chanakyan Sar avan and Babu. each plays one outdoor and one Indoor game The outdoor games are Volleyball, Cricket, Badminton and Hockev and the Indoor games are Chess, Carr oms C"ards and Table Tennis Ct anakyan does not play carr oms The Hockey player plays cards too Babu. does not play Hockey or any other game starting with (C) Saravan does not play Hockey and Volleyball The first letter of a certain person" Name IS the same. as the games played by him

211 Chankyan plays

1} Cricket and Chess

2) Cricket and Cards

3) Cricket and Carroms

4) Cricket and Table Tennis

22) The cards are played by

1) Babu 7.1 Han 3) Saravan a) None

23) Carroms are played by

1) Babu 2) Ohenakyan

3) Saravan 4) None

24) Babu plays 1) Hockey

3) Badmmton

2) Cricket

4) Vol'ey Ball

25) Saravan plays

1) Badminton and Carroms

2) Volleyball and Table Tennis

3) Hockey and Cards

4) Cricket and Chess

DIRECTIONS:

Four tnends Raman, Narayan, Pavan and Sundaram are Lecturer, professor, Reader an Engineer but nol in the same order and are interested in singing, Violin, Veena and Drama, but not in the same order

78

Narayan a Reader is not interested in Veena Raman is interested in Drama but IS not an Engineer Lecturers and professors are not interested in Violin Pavan IS not Interested In Singing nor he IS concerned With Lectureship nor Professorship Sundaram IS interested in Singing and has nothing to do with professorship The Veena Vocalist IS an Engineer too

26) Who is a singer
1) Pavar 2) Raman
3) Narayan 4) Sundaram
27) Who IS an Engineer?
1) Sundaram 2) Pavan
3) Raman 4) Narayan
28) Who IS a Dramatist
1) Pavar 2) Narayan
3) Raman 4) Sundaram 29) What IS Sundaram's Profession?

1) Professor 3) Reader

2) Lecturer 4) Enqmeet

30) Reader's post IS occupied by

1) Raman 2) Narayan

3) Sundaram 4) Pavan

DIRECTIONS:

Englishman and French are polinctans. Poets and essayists French and Germans are Politicians Es sayrsts and musrcrans. Germar-s and Russians are politicians, academicians and rnustcrans Indians and Russians are ac ade rnicr an s , poets and rnustcrans Indians and Englishmen are essayists poets and acadenucrans

31) Name the people who are poets, academicians. musicians and es s ayrsts

a) Indians b) Russians

C) Germans d) French

e) Englishmen

32) Name the people who ale politicians, academictans. musicians and esaaytsts

a) French b) Germans

c) Russians d} Indians

e) Englishmen

33) Name the people who are musicians, poets, politicians and essayists

a) Indians b) French

c) Germans d) Russians

e} Englishmen

34) Name the people who are politicians, academicians, essayists and poets

a) Engllshmen b) Indians

c) Russians d) French

e) Germans

35 Name the people who are musicians poets academicians and politicians

a) Russians b) French

c) Indians d) Germans

e) Englishmen

DIRECTIONS,

Five children Han Babu Christopher Sridhar and Thomas are Interested In hockey, Tennis, Chess Badrmntn and SWimming, but not Ir, the same order Babu and Christopher are not interested in Swimming, Thomas ruetne- plays hockey nor Chess Hari and Sridhar do not play chess, Chnstopher does not play HOCKey Han and Thomas do not play Badmmton The first letters of the names and the games played by that person IS not the same

36) Babu plays a) Chess

c) Badminton

b) Hockey d) Tennis

37'1 Who is interested 111 SWimming

a) Babu b,l Hall c) Thomas

d) Christopher e) Sridhar

38) Who plays Tennis a} socre.

c) Han

b) Babu

dl Thomas

39) Badminton IS played oy

a) Christopher b) Babu

C) Sndhar d) Han

40) Who plays hockey a) Sridhar

c) Hari

b) Thomas d) Babll

DIRECTIONS:

Four persons Reje s h Arun. Mohall and Ganesh are married to Anuradha, Nahm. Kamala and Maduri but not in the same order Each couple has a car and the colours are Green, Black, Red and White but not In t-ie same order, Nahru and Mohan do not have Green car nor they are married Anuradha and Rajesh do not have a White car, Maduri has a Red car ard Kamala does not have a White car. Mohan and Rajesh do not have a Green car. Ganesh IS not madvrf's husband. The white car belongs to Anand. Anuradha IS the wife of Mohan

41) Maduri is the wife of

a) Rajesh b 1 Ganesh

c) Mohan d) Arun

42) Who is having a Black car

a) Mohan and Anuradha

b) RaJesh and MadUri

c) Kamala and Ganesh

d) Nalini and Arun

43) Rajesh has . __ ~ car

a) Black b) Green

c) Whde dl Red

44) Green car belongs to
a) Rajesb b) Ganesh
C) Arun dl Mohan
451 Arun IS the husband of
a) Madan b) Kamala
c. Nalmi dl Anuraoha DIRECTIONS:

Mrs Radha has wool In three colours Red Black and White, she wants to knit sweaters for her husband, her son and her brother, uSing two colours for eacf sweater One of them IS fair one 15 tat and one IS black

Mr Knshna, the husband of Radha IS not fair and one of the coro.rrs In his sweater IS gOing to be white Mrs Radhas brother IS not brace and one Of the colours In his sweate- would be red Radha's son is not tall and vdl not have black colour In t-rs sweater. Tile fair person will have red and black colours In his sweater

46\ What are the colours used In Kns hnas

sweater
a) Black and Red D) Black and White
c) Red and While d) none
47) Who IS fair
a) Son b) brother
c) husband d) none
48) The son IS
a) Fair b: Dark c) Tall none 79

lJirectians far Q.No.1·5:

Five executives of a European corporation hold a conference in Rome

Mr A convercses In Spanish and Italian Mr B converses in Spanish and English Mr C converses in English and Italian Mr 0 converses in French and Spanish

Mr E a native Italian, can also converse in French

Which, of the fOl/owing, can act as interpreter when Mr Cand Mr D wish to confer?

a) onlyMr A c) only Mr E

e) Any of the other three executives

b) only MrB d)MrAorMrB

WhIch of the following cannot converse without an interpreter?

a) MrS and MrE c) Mr.A and Mr.C e) Mr A and Mr.E

b)MrAandMrS d) MrS and MrD

Besides Mr E, which of the following can converse with Mr. D without an interpreter?

a) only MrA c) only Mr C

e} Messers A.B, and C

b) only Mr.B

d) Messers A and S

If a sixth executive is brought In, to be understood by the maximum number of the orqmal five, he should be fluent In

a) English and French C) French and italian

e) English and Spanish

b) italian and English d) Italian and Spanish

Of the languages spoken at this conference, which are the.two least common?

a) English and Spanish c) Italian and Spanish e) French and Spanish

b) English and French d) English and Italian

Directions for a.No. 6 ~ 11:

At a congress of the progressive Federal party. the seven top party leaders. who are all cabinel ministers, are seated on the platform In order of rank. The Prime Minister, the party leader. is in the centre. The closer a person is to the Prime Minister. the higher is his or her rank, with a person on the Prime Minister's right out ranking one equidistant from the Prime Minister on her left, The seven leaders are Arning, Brenner, CivH'I, Dorner. Eckland. Fentz and Grell

Frenz is four places to the left of the Minister of Agriculture, who is two, places to the r'lght of C1vlli, Brenner's neighbours are Arning and the Minister of Agriculture. Grell is two places to the left of Dorner

The Ministers of Education. Mining and Culture are seated together in that order. from left to right

The remaining ministers are those of Social Welfare and Defense.

80

The Minister of Culture is

a) Arnlng d) Dorner

b) Brenner e) Eckland

c) Civl!i

The fifth- ranking person in the party hierarchy IS

a) Grell, the MlnisterofMrning

b) Fentz, the minister of culture

c) Dorner, the Prime Minister

d) Eckland, the Minister of Defense

e) Arninq. the Minister of Education

The Minister of Social Welfare

I Outranks the Minister of Mining

II. is outranked by the Minister of Mining

a) I only

c) I and II only

e) Neither I nor II

b) II only

d) I or II, but not both

How many of the seven party leaders outrank the Minister of Education?

a) d)

b) e)

c)

1) If, durinq the congress, the Minister of Aqriculture and the Minister of Education are ordered to exchange positions, which IS true?

a) Arming will move to a seat six places away from nrs original seat

b) Fentz Will move up five places in the leadership ranking

c) E:kland will move to a seat three places away from his original seat

d) Grell will move up four places In the leadership ranking

e) Eckland wm move from the prime Minister's left side to hrs nght

11 If, dunng the congress. Eckland IS demoted two places in the party leadership ranking, which is true?

a) The Minister of Defense moves up one place In the leadership ranking

b) CiVil becomes the second - ranking leader in the party

c) The Minister of Mining moves up two places In the leadership ranking

d) Dorner IS demoted within the leadership

e) The positions of five persons within the leadership remain unchanged

Directions for Q.No.12 -18:

For a panel of professors to assess the State of the Union Message on public TV, the producer must choose two Republicans and two Democrats At least one professor must be an economist and at least one a military expert. Available Republicans are Abbott. Bartlett, Catlett. Dorset, and Everett, available Democrats are Fawcett, Gantlet, Helfet, and lnsett. Catlett, Fawcett and Ganglet are economists. Dorset and Insett are military experts Fawcett will not sit in the same room with Catlett. and will take part only if Abbott IS on the panel Derset refuses to take part with Gauntlet, adn Everett refuses to take part with Insett

81

12 Which of the following IS not acceptable panel?

a) Fawcett, Helfet, Abbott, Dorset

b) Fawcett, lnsett, Abbott, Dorset

c) Gantlet Hellet Abbott, Catlett

d) Gantlet, Insett, Abbott Catlett

e) Helfet, Insett, Bartlett. Carlett

13 If Abbott and Bartlett are chosen as the Republicans, who can be chosen as the Democrats?

a) Fawcett and Insett only

b) Fawcett and Insett or Gantlet and Insett only

c) Fawcett and Gantlet or Gantlet and Helfet only

d) Fawcett and Insett, Gantlet and Insett, or Helfet and Insett

e) Gantlet and Helfet. Gantlet and Insett. or Helfet and Insett.

14 If Gantlet is chosen, which of the following must be true?

Any acceptable panel must contain Insett Any acceptable panel must contain Abbott

III There is no acceptable panel which contains Bartlet.

a) c) e)

I only

land II only 1.11 and III

b) d)

II only

II and III only

15 How many acceptable panels can the producer put together?

a) d)

b) e)

7 10

c)

16 Which of the following pairs cannot be part of an acceptable pane! ?

Fawcett and Gantlet
Bartlett and Dorset
III Catlett and Dorset
a) lonly b) III only
c) I and II only d) II and III only
e) I,ll and III 17 Which Republican belongs to the smallest number of different acceptable panel?

a) d)

Abbott Dorset

b) e)

Bartlett Everett

c)

Catlett

18 Which professor belongs to the greates number of different acceptable panels?

a) d)

Abbott Hellet

b) e)

Bartlett Insett

c)

Gantlett

Directions for Q.No. 19-22

At a found dinner for eight. the host and hostess are seated at opposite ends of a rectangular table, with three persons along each side, Each manmust be seated next to at least one woman, and vice-versa

Allan is opposite Diane, who is not the hostess

George has woman on his nqht and is opposite a woman

Helga IS at the hostess's nght next to Frank,

One person is seated between Belinda and Carol

19 The eight person present, Eric, must be

the host

II scated to Diane's right

III seated opposite Carol

a) c) e)

I only

I and II only 1.11 and III

b) dl

III only

II and III only

20 If each person IS placed directly opposite his or her spouse which of the following pairs must be married?

a) George and Helga

b) Belinda and Frank

c) Carol and Frank

d) George and Belinda

e) Eric and Helga

21 Which person is not seated next to person of the same sex?

a) c) e)

Allan Carol Eric

J) j)

Belinda Diane

22 George IS bothered by the cigarette smoke of his neighbour and exchanges seats with the person four places to his left, Which of the following must be true following the exchange?

No one is seated between two persons of the opposite sex

One side of the table consists entirely of persons of the other sex,

III. Either the host or the hostess has changed seats,

a) c) e)

I only

I and II only Neither I,ll not III

b) d)

III only

II and III only

63

Directions for Q.No. 23 to 24:

Observance of Memorial Day, which falls on a Saturday this year, will be as follows for the tristate area (New Albion. New Shetland, New Wales)

BankS and government departments which are normally open on Saturdays will close

Those normally closed on Saturdays will close as follows:

Bank will close on Fnday In New Wales and Monday In New Shetland.

State government offices will close on Fnday In New Albion and New Shetland

Sanitation pick-up In Monday·Wednesday-Friday pickup areas will be cancelled Friday In New Albion and New Shetland. and Monday in New Wales: pick up in Tuesday- Thursday-Saturday areas will be cancelled Saturday in all three states

The post office and other federal offices. nonmally open on Monday through Saturday will be closed on Saturday but open on Friday and Monday in all three states

(Banks are normally open on Saturday only in New Albion; State government offices are normally opened on Saturday only in Wales

23 Mrs. Semkow goes to the post office, the bank, and the state income tax bureau on Monday She may live In

New Albion
III New Wales
a) lonly
c) lor III only
e) 1.llorlli II. New Shetland

b) II only

d) liar III only

24 In which area(s) IS there no deviation from normal service on Monday for any of the services listed?

a) All of new Albion

b) Monday·Wednesday-Fnday pickup areas In New Albion and Wales

c) Tuesday-Thursday-Saturday pickup areas In New Shetland and New Wales.

d) All of New Wales

e) Mondays·Wednesday-Friday areas in New Shetland

Directions for Q.No.25 to 30:

ML Pesth. forman for Buda Construction Co. is hiring five persons to do wiring' and plumbing on a site. He must have a minimum of two electricians. Nine persons are sent by the union hiring hall Mike, Nice, and Olive are electricians. while Rich, Steve, Tom, Ulysses, VIC, and Wassiyare numbers.

Pesth IS unwilling to hire Ulysses and V'c together, because he known from past experience that they fight all the time

Steve and Tom are buddies and will only work together.

Ol.ve won't work with Rich, a Moravian, because she despise Moravians,

84

25 If Mike. Nick, and Olive are hired. the team of plumbers can consist of

a) Steve and Tom only

b) Steve and Tom or Ulysses and vrc

c) Ulysses and Wassily or Vic and Wasslly

d) Steve and Tom, or Ulysses and Wasslly or Vic and Wasslly

e} Steve, Tom, and either Ulysses, Vic or \/Vassley

26. Pesth has the greatest number of choices for hiring as plumbers if the electricians he chooses are

a) Mike, Nick, and Olive

b) Mike. and Nick

c) Mike, and Olive

d) Nick and Olive

e) Either Mike or NiCK. plus Olive

27 If Rich is hired, the other persons hired must be

a) Mike, Nick Steve. and Tom

b) Mike Nick, Olive and either Ulysses VicorWassily

c) Mike, and Nick. togelher with either Steve and Tom or Ulysses and Wasslly

d) Mike, and nick, together with either Ulysses and VIC or Vic and Wassily

e) Mike and Nick, together with either Steve and Tam Ulysses and Wassily or Vic and wasslv

28 Pesth can put together the rest of his crew in the greatest number of different ways If the hires

a) ci e:

Steve and Tom Ulysee Wassily

b) d)

Olive VIC

29 If Mike is hired and Nick IS not, which of the following statements must be true?

Steve and Tom are hired

Either Ulysses or Vic is hired. but nor both

al c: e)

I only

either I or II. but not both neither I no- II

b) d)

II only

both I and II

30 Which of the following statements must be true?

If only two electricians are hired, the plumbers must include Steve and Tom If Olive is not hired. Rich must be hired.

III If either Mike or Nick IS not hired, Steve and Tom must be hired

al C) e)

I only III only

II and III only

b) d)

II only

I and III only

Directions for a.No, 31 to 33:

Byram and Adoniram are code clerks at the Pentagon. They Me also secret agents for foreign governments One IS in the pay of the Sulqravians and the other is in the pay of the Caroflnqains. If a document IS stolen, it will take four days to reach the Sulgravian government and five days to reach the Carolingian Government

85

Byram is given top-secret documents to encode on October19 and 22, Adoniram IS given a top-secret document to encode on October 21. Byram and Adorurarn have lunch together on October 20

Agent of foreign governments do not transmit documents directly to governments that do not employ them but may sell documents to an agent of another government. An agent who transits a document always does so, on the day he receives it.

31 If Adonirarn IS working for the Sulqravians. the Sulgravian government may receive documents on

III a) c)

October 24 October 26 I only

I and II only

October 25

b) d)

III only

II andilionly

32 A top-Secret document IS received by the Carolingians on October 25. It could have been

a) stolen and transmitted by Byram

b) stolen and transmitted by Adoniram

c) stolen by Adoniram and sold to Byram, who transmitted it

d) stolen by Byram and sold to Adoniram, who transmitted it

e) stolen by either Byram or Adoniram and soldto the other, who transmitted it

33 If Adoniram is working for the Carolingians, which must be true?

a) The Sulgravians may receive documents only on October 23

b) The Caroliqians may receive documents only on October 26

c) The Sulgravlans may receive documents only on October 24,26,27

d) The Carollglans may receive documents only on October 24.25, 26.

e) No documents received by the Sulgravians can have been bought at Byram and Adoniram's lunch time meeting

Directions for a.No. 34 to 40;

The organizer of Local 58 of the hospital workers is forming a five - person team to leaflet a nearby hospital. The team must contain two persons to distribute leafiets, one speaker to address the workers who stop, and a two person defence squad. A, Band C are possible leaf letters; C,D and E are possible speakers, F, G and H are possible members of the defense guard. A and C prefer to work with each other on the same team E prefers to work only if F works

34 Which choice of personel is impossible if all preferences are respected?

a) A and B as leafletters, C as speaker

b) Band C as leafletters

c) A and C as leafletters, F and H on defense

d) Either D or E as speaker, with F on defense

e) G and H on defense.

35 If A and Bare leatletters and all preferences are respected, which is (are) true?

I. C IS the speaker.

II. F IS on defense.

III. Either F or G IS on defense.

86

a) c) e)

I only III only

I and III only

b) d)

II only

I and II only

36 Which is a possible team If all preferences are respected?

a) c) e)

A,B,CD,F AB,C,F,G B.CD.F.G

b) d)

A.C,D,EF ACE,G,H

37 If A IS chosen as a member of the team and all preferences are respected which must be true?

a) B must be leafletter

b) C must be a leafletter

c) F must go

d) Any of the three defense personnel may go

e) Neither D nor E can go.

38 How many different possible teams can the orgamzer assemble, 'f all preferences are respected?

a) d)

13

b) e)

c)

15

39 Wh,ch person must be chosen as part of any team, if all preferences are respected?

A

a) c) e)

I only

I and II only I, Ii and III

b) d)

III only

II and III only

40 Which person can be part of the smallest number of different possible teams, if everyone's preferences are respected?

a) d)

A D

Directions for Q.No. 41 to 42:

b) e)

B E

c)

C

Delegations from Wallachla and Rumelia are meeting to discuss military, trade, and oip'crnauc problems Each delegation consists of a cnarperson two military attaches, and two trade experts The Wallchlan delegation consists of A,B,C,D and E. the Rumelian delegation of F,G,H,I and J. Each chariperson is to occupy the middle seat in a row of five on each of two sides of a rectangular table,

1) A insists on being seated at the opposite end of the table from B,

2) G, who is deaf in his fight ear, must be at the fight end of the table

3) Neither D nor F is chairperson,

4) The Wallach Ian military attaches, one of whom is 8, are seated together and neither is opposite either of the Rurneuan military attaches, neither of whom is G

5) C, a trade expert, IS seated opposite H

41 F may be a

a) Trade expert seated next to I

b) Military attache seated next to I

c) Military attache seated next to J

d) Trade expert seated next to H

e) Trade expert seated opposite 8

42 If J is a military attache, which of the following must be true?

The Rumelian chairperson is I F is a trade expert

III I IS a trade expert

a) I only

b) II only

c) I and II only

d) lor III, but not both

e) II or III, but not both,

Directions for Q.No. 43 to 46:

1) A causes B or C, but not both

2) F occurs only if B occurs,

3) D occurs onlu if 8 or C occurs

4) E occurs only if C occurs,

5) J occurs only if E or F occurs

6) D causes G or H or both

7) H occurs if E occurs

8) G occurs if F occu rs

43 If a occurs, which may occur?

F and G

a) I only

b) II only

c) III only

d) II and III but not both

e) 1,llandlll

44 If B occurs, which must occur?

45.

a) E

c) D

e) J

If J occurs, which must occur?

a) c) e)

E

Either 8 or C Both 8 and C

D

II.

A

68

Eand H

III.

D

b) d)

Both E and F GandH

b) d)

Both E and F G

III.

F

a) c) e)

I only

I and II only 1.11 and III

b) d)

II only

II and III only

Directions for O,No, 47 to 53:

Joe, Larry, Ned, Mary, Paul, Willy, Crystal, Albert, Bob, Frank, Ellen and Rick all live in the same SIX -floor building_ There are two apartments per floor. No more than two persons live in any apartment Some apartments may be empty

Larry and his room mate live two floors above Albert and his room mate. Crystal.

Joe lives alone, three floors below Willy and two floors below Ellen Mary lives one floor below Albert and Crystal

Ned lives three floors above the floor on which Bob and Frank have single apartments.

Rich and Paul live In single apartments two floors below Mary,

47 Which of the following lists the persons named In the correct order, gOing from the bollom floor to the tcp?

a) Rick, Bod, Marry Albert, Larry, Ned

bJ Rick, Frank, Ned, Ellen, Larry, Crystal

c) Paul, Bod, Joe, Crystal. Ned, Larry

d) Larry, Ellen, Albert, Mary, Frank, Rick

e) Larry Joe, Mary, Albert, Bod, Rick

48 Which of the following pairs must live on the same floor?

Ned, Ellen

Joe, Mary

III

Albert, Larry

a) c)

I only

I and II only

b) d)

III only

II and III only

49 Larry's room mate, assuming that it is one of the persons mentioned, is

a) c) e)

Ellen Mary Paul

b) d)

Willy Ned

50, Rick lives on the

aJ first floor, below Bob or Frank

bJ second floor, below Joe or Albert and Crystal

c) third floor, above Mary or Ellen

d) fourth floor, opposite Albert and Crystal

e} sixth floor, opposite Larry and hi. roommate

51, An empty apartment or empty apartments may be found on the

a) c) e)

second fioor only b)

fifth floor d)

fourth or sixth floor or both

fourth floor only

third or sixth floor, but not both

89

52 Joe arranges to move into an apartment two floors down. whose occupant moves into an apartment one floor up. The occupant of this apartment moves into one three floors, up whose occupant takes Joe's old apartment The new occupant of Joe's old apartment is

a) c) e)

Bob or Frank Mary

Paul

b) d)

Ned or Ellen Rick

53 Dorothy lives with a roommate. Her room mate could be any of the following EXCEPT

a)

Willy

b)

Mary

c)

Frank

Directions for Q.No. 54 to 59:

The letters, A.8.C,Q,E,F and G not necessarily in that order, stand for seven consecutive integers from 1 to 1:

Q is 3 less than A

B IS the middle term

F IS as much less than B as C is greater than 0 G IS greater Ihan F

54 The fifth Integer IS

a) d)

A E

b) e)

C F

c)

D

55 A is as much greater than F as which integer is less than G ?

a) d)

A D

b) e)

B E

c)

c

56 If A = 7. the sum of E and G IS

a)
d) 14
57 A . F =?
a)
d) b)

10

c)

12

e) 16

b) c)

e) cannot be determined

58 An integer T is as much greater than C as C is greater than E. T can be written as A +E. What IS O?

a) d)

4

b) e)

c) cannot be determined

59 The greatest possible value of C IS how much greater than the smallest possible value of 0 ?

a) d)

b) e)

c)

90

Directions for Q.No. 60 to 64:

A project to consolidate the programs of a large university and a small college is set up. It IS agreed that the representatives work in small committees of three, with two representatives of the large university. It was also agreed that no committee be represented by faculty members of the same subject area The large university was represented by the following professors J, who teaches English literature. K, who IS chairman of the Mathematics Department, and L, who IS In the Department of Natural SCience The small College appointed follOWing M. who teaches mathematics, N, who IS a Latin teacher, and 0 and P who teach English literature

60 Which of Ihe following must be true?

If J serves on a committee, P must be assigned to that committee

If J cannot serve on a committee, then M cannot be assigned to that committee

III If J cannot serve on a committee, then L must serve on that committee.

a) d)

I only

I and II only

b) e)

II only c)

II and III only

III only

61 Which of the following represents a committee properly composed?

a) d)

K,L,N J.O.N

b) e)

K,L,M J,K,M

J,KL

c)

62 Which of the following may serve with P

a) c)

K and M K and 0

b) d)

K and L J and K

e) M and N

63 If L is not available for service, which of the following must be on the committee?

a) d)

M and J N and 0

b) e)

o and J P and J

c)

Nand J

154 Which of the tollowinq must be true?

Nand 0 are always on the same committee M and 0 never serve on the same committee

III When M serves L, must serve

a) c) e)

I only

I and II only II and III only

Directions for Q.No. 65 to 69:

b) d)

II only III only

The office staff of XYZ Corporation presently consists of three book keepers. (A.B and C) and five secretaries (D,E,F,G and H) Management IS planning to open a new office in another city using three secretaries and two book keepers of the present staff. To do so they plan to separate certain individuals who do not function well together

The following guidelines were established to set up the new office

91

Book keepers A and C are constantly finding fault with one another and should not be sent as a team to the new office

2 C and E function well alone but not as a team, They should be separated.

3 0 and G have not been on speaking terms for many months. They should not go together

4. Since 0 and F have been competing for promotion they should not be team.

65 If A IS to move as one of the book keepers. Which of the following CANNOT be a possible working Un!1 ,

a] c) ej

ABDEH f.BEFH ABFGH

b) d)

ABOGH ABEGH

66 If C and F are moved to the new office, how many combinations are possible?

a) d)

b) e)

2 5

cj

67 If C IS sent to Ihe new office which member of the staff CANNOT go with C?

aj d)

B G

b) e)

o H

cj

68 Under the gUideline developed which of the following MUST to be new office?

a) d)

B G

b) e)

o H

c)

E

69 If 0 goes to tile new office which of the follOWing is (are) true?

C cannot go A cannot go
H must also go
a) I only b) II only
c) I and II only d) I and III only
e) l.lland III Directions for Q.No. 70 to 73:

To obtain a government pomt In the Republic of Malabar, you must either be a member of the ruling Independence party or a personal associate of President Zarnir

Party members seeking a government post must either give a substantial donation In gold bullion to the party's campaiqn

Fund or make a televised speech denouncing President Zamir's political enemies

Gold bullion may be purchased only at the National Bank. which does business only with those who have beer. certified as politically sound by the Minister of Justice.

Only those who either have been certified as politically sound by the Minister of Justice or have donated 300 hours of service to the Independence Party are allowed to make political speeches on television.

92

To become a personal associate of President Zamir, you must either give a substantial donation In gold bullion to the president's personal expense account or perform personal services for a member of his immediate family

Before appointing a personal associate to a government post, President Zarrur always checks to make sure that he or she has been certified as politically sound by the Minister of Justice

70 Mr Mizar IS member afthe Independence Party. To obtain a government post, his next step must be to either

a) Be certified as pOftticallysound by the Minister of JUstice, or give a substantial donation In gol J bullion to the party'. campaign fund

b) Donale 300 hours of service to the Independence Party, or give a substantial donation in gold bullion to the president's personal expense account.

c) Be certffied as politically aound by the MinISter of Justice, or donate aOO hours of services to the party

d) Perform personal services for a member of President Zamir's immediate family, or make a televised speech denounCing the president'spalitical enemieS

e) Be certifiedpoi~ically sound by the Minister of Justioe, or.beoome ",Personal associate of Pne~Zimlr,

71 All those who wish to obtain government post must

become personal associates of President Zamir

be certified as politically sound by the Minister of Justice.

III purchase gold butlion at the National Bank

a) c) e)

I only III only

neither I,ll and III

b) d)

II only

II and III only

72 Mr. Razim has been certified as politically sound by the Mintsterof Justice. He may obtain a government post immediately only If he

a) has donated 300 hours of services to the Independence Party

b) is allowed to make political speeches on television

c) is a member of the Independence Party

d) is a personal associate of President Zamir

e) has purchased gold bullion at the National Bank

Because of financial crisis, the National Bank is closed indefinitely Those who wish to obtain government posts during this period must

a) either perform some kind ofservice or make televised speeches de -Iouncing President

Zamir's political enemies

b) become members of the Independence Party

c) donate 300 hours of service to the Independence Party

d) become personal associates of President lamir

e) either become members of the Independence Party Dr perform any services for the party

93

Directions for Q,No, 74 to 77:

Professor Kittredge literature seminar includes students with varied tasks in poetry. All those in the seminar who eruoy the poetry of Browning also enjoy the poetry of Eliot Those who enjoy the poetry of Eliot despise the poejry 01 Colendeqe

Some of those who enjoy the poetry of Eliot also enjoy the poetry of Audan

All those who enjoy the poetry olColeridege also eruoy the poetry of Donne,

Some of those who enjoy the poetry of Donne also enjoy the poetry of Eliot Some of those who enjoy the poetry of Auden despise the poetry of Colerideqe.

All those who enjoy the poetry of Auden despise the poetry of Frost

74 Miss Garfield enjoys the poetry of Donne Which afthe following must be true?

a) She mayor may not enjoy the poetry of Coleridge

b) She doe not enjoy the poetry of Brownrng,

c J She enjoys the poetry of Auden.

d) She does not enjoy the poetry of Eliot

e) She enjoys the poetry 01 Colerrdge

75 Mr Huxtabel eruoys the poetry of Browning. He may also enjoy any of the following poets except

a) c)

Auden Donne

b) d)

Coleridge Elrot

e)

Frost

76 MIss Inaguch enjoys the poetry of Coleridge, Which of the follOWing must be false?

a) She does not enjoy the poetry of Auden

u) She enjoys the poetry of Donne

C) She enjoys the poetry of Frost

d) She does not enJoy the poetry of BfDw,ling

e) She may eruov the poetry of Eliot.

77 Based on the information provided, which of the fonowing statement concerning the members of the seminar must be true?

a) All those who enJoy the poetry of Eliot also enJoy the poetry 01 Browning

b) None of those who despise the poetry of Frost enjoy the poetry of Auden

c) Some of those who despise the poetry of Auden despise the poetry of Coleridge

d) None of trice who enjoy the poetry of Browning despise the poetry of Donne,

e) Some of those who enjoy the poetry of Frost despise the poetry of Donne,

Directions for Q,No, 78 to 81:

1) All G's are H's
2) All G's are J's or K's,
3) All J's and K's are G's
4) All L's are K's
5) All N's are M's,
6) No M's are G's 94

S-ar putea să vă placă și